Dermatology Flashcards

(323 cards)

1
Q

Define atopic dermatitis?

A

An inflammatory skin condition characterised by dry pruritic skin, with a chronic relapsing course, occurring predominantly in children.

How well did you know this?
1
Not at all
2
3
4
5
Perfectly
2
Q

What are the RFs for atopic dermatitis?

A

FHx, allergic rhinitis, asthma, atopy, <5yo

How well did you know this?
1
Not at all
2
3
4
5
Perfectly
3
Q

What are the signs and symptoms of atopic dermatitis?

A
Dry (xerosis) pruritic skin
Hypopigmentation
Excoriations 
Scaling 
Papules/vesicles, erythema 
At borders of joints in children mainly
Lichenification (end stage)
How well did you know this?
1
Not at all
2
3
4
5
Perfectly
4
Q

What investigations are performed for atopic dermatitis?

A

Usually no need - simply clinical
IgE Raised
Allergy testing
Skin biopsy

How well did you know this?
1
Not at all
2
3
4
5
Perfectly
5
Q

What is contact dermatitis?

A

An allergic or irritant inflammatory skin reaction caused by contact with an external agent.

How well did you know this?
1
Not at all
2
3
4
5
Perfectly
6
Q

What are the RFs?

A

Atopic dermatitis, atopy, occupational exposure (ie Nickel sulfate)

How well did you know this?
1
Not at all
2
3
4
5
Perfectly
7
Q

What are the signs and symptoms?

A

Prev episodes, repeat symptoms on exposure, hands and face, no involvement of concealed places, dry skin (xerosis), pruritus, erythema, persistence, scaling, burning sensation, milliaria (prickly heat), lichenification, vesicles/bullae

How well did you know this?
1
Not at all
2
3
4
5
Perfectly
8
Q

What are the investigations for contact dermatitis?

A

Patch testing - inflammatory reaction graded from 1-3, within 2-7 days
Skin biopsy

How well did you know this?
1
Not at all
2
3
4
5
Perfectly
9
Q

Define urticaria?

A

Erythematous, blanching, oedematous, pruritic lesions that resolve within 24hrs and leave no residual markings on resolution

How well did you know this?
1
Not at all
2
3
4
5
Perfectly
10
Q

What are the RFs for urticaria?

A

Fhx
Drug triggers - NSAIDS, penicillins, muscle relaxations, diuretics, sulphonamides
Food triggers - milf proteins, eggs, shellfish, Fin fish, peanuts
Insect bites
Viral infections
Weak - female

How well did you know this?
1
Not at all
2
3
4
5
Perfectly
11
Q

What are the signs and symptoms of urticaria?

A

Erythematous, blanching, oedematous, pruritic lesions, last less 24Hrs, no residual markings, associated Angioedema (swelling of deep layers of the subdermis)
Potential stridor

How well did you know this?
1
Not at all
2
3
4
5
Perfectly
12
Q

What are the investigations performed into angioedema?

A
FBC with differential
U+Es 
ESR and CRP 
AntiIgE antibody testing 
antithyroid antibody - +ve in hashimotos 
TSH - Raised if underlying thyroid 
ANA  - +Ve in rheumatic 

SKIN BIOPSY

How well did you know this?
1
Not at all
2
3
4
5
Perfectly
13
Q

Define BCC?

A

A common skin neoplasm of predominantly older populations associated with previous intense sun exposure. It causes local destruction but rarely metastasises.

Clinically presents as a pearly white papulo-nodule or firm plaque.

Patient usually has prominent solar damage or a history of considerable ultraviolet (UV) exposure.

Can be locally aggressive, but rarely metastasises.

Biopsy with histopathological evaluation is essential; findings are of dermal masses of varying sizes and shapes composed of basophilic cells with large oval rather uniform nuclei and scant cytoplasm. These masses exhibit a peripheral cell layer demonstrating a palisading pattern of nuclei.

Diagnosis of a cancer is histological; treatment varies.

Complications are unusual if this cancer is treated adequately.

How well did you know this?
1
Not at all
2
3
4
5
Perfectly
14
Q

What are the RFs for BCC?

A

SAXU
Sunlight, arsenic, X-rays and UV rays

Xeroderma pigmentosum, basal cell naevus syndrome
Transplants

How well did you know this?
1
Not at all
2
3
4
5
Perfectly
15
Q

What are the S+S associated with a BCC?

A

Pearly papule with associated telangectasias
rolled borders
Non-healing ulcers/wounds

Rare = metastasis

How well did you know this?
1
Not at all
2
3
4
5
Perfectly
16
Q

What are the investigations for a BCC?

A

Skin biopsy - hyperchromatic cells with high nuclear:cytoplasmic ratio, fibrinous surrounding stroma

How well did you know this?
1
Not at all
2
3
4
5
Perfectly
17
Q

Define erythema multiforme?

A

A typically acute self limiting but often relapsing mucocutaneous inflammatory condition. It is a T4 hypersensitivity reaction to certain infections/drugs/vaccinations.

How well did you know this?
1
Not at all
2
3
4
5
Perfectly
18
Q

What are the RFs for erythema multiforme?

A

Strong - M pneumoniae, HSV, Hx
Weaker:
Drugs - statins, lidocaine, aminopenicillin, tnf-a, NSAIDs, antimalarials, anticonvulsants
Viral - EBV, CMV, HEP B (and vaccine), histoplasmosis

How well did you know this?
1
Not at all
2
3
4
5
Perfectly
19
Q

What are the signs/symptoms for erythema multiforme?

A

Targetoid lesions on extremities, targetoid lesions, clustered vesicles on erythematous bone, Mucosal erosions, Recurrence, Rapid onset, Last >24hrs, heptalomegaly, red tympanic membrane

How well did you know this?
1
Not at all
2
3
4
5
Perfectly
20
Q

What investigations are undertaken for erythema multiforme?

A

Usually a clinical diagnosis. If severe infection -> investigate causes.

Ie 
FBC - WCC
U+Es - possible volume dep
HEP B serology
M.pneumoniae titre
HSV serology

Stopping any relevant medications

How well did you know this?
1
Not at all
2
3
4
5
Perfectly
21
Q

What is erythema nodosum?

A

A common cutaneous hypersensitivity reaction consisting of erythematous, tender nodules on the shins.

How well did you know this?
1
Not at all
2
3
4
5
Perfectly
22
Q

What are the RFs, and what is the main cause of erythema nodosum?

A

RECENT STREP INFECTION = most common

Others - TB, histoplasmosis, coccidiodomycosis, sarcoidosis, blastomycosis, leprosy, Behcets

How well did you know this?
1
Not at all
2
3
4
5
Perfectly
23
Q

What are the signs / symptoms of erythema nodosum?

A

Painful erythematous nodules on the shins,
Nodules elsewhere, Joint pains, uveitis, fever,
Candle wax dripping on the retinal veins

How well did you know this?
1
Not at all
2
3
4
5
Perfectly
24
Q

What are the investigations that confirm erythema nodosum?

A
Typically just CLINICAL 
Can investigate causes
ASO titre - recent strep infection +ve
TB tests if suspect
ESR WCC increased
How well did you know this?
1
Not at all
2
3
4
5
Perfectly
25
What is a lipoma?
A slow growing benign mesenchymal tumour that forms a lobulated lesion composed of adipocytes surrounded by a thin fibrous capsule.
26
What are the RFs for a lipoma?
Genetic trauma alcohol
27
What percentage of the population have lipomas?
1%
28
What are the signs/symptoms/features of a lipoma?
<5cm, discrete, soft (SMOOTH BUT TENSE MAY SUGGEST CYST), mobile, untethered, rarely painful, uncomfortable beneath tight/irritant clothing, SUPERFICIAL (if deep rule out liposarcoma) Uncommon - GI bleed/obstruction, weakness, parasthesia,
29
What investigations should be performed with lipomas?
MRI/CT - see discrete lobular lesion with no/little septa and a thin capsule Core needle biopsy / excisional biopsy - histology Upper GI contrast if suspect blockage
30
Define melanoma?
A malignant tumour arising from melanocytes, most common in young adults and presenting as a new/changing deeply pigmented skin lesion.
31
What are the RFs for melanoma formation?
Fitzpatrick T1/2, blonde/red hair, >50 Naevi, hx atypical naevi, Fhx, personal Hx, sun exposure, sun bed use, light eye colour, xeroderma pigmentosum, large congenital naevi, immunosuppression, increased freckle density
32
What are the signs/symptoms of melanomas?
ABDCE - asymmetry, border irregularity, diameter >6mm, colour variability (light/dark), EVOLUTION 'Ugly duckling', highly pigmented, fixed lymphadenopathy, constitutional symptoms, bluish white vale, melanoma striata (DARK band nail).
33
What are the relevant investigations for a melanoma?
``` Dermatoscopy Skin biopsy MRI/CT - lung/spinal mets Sentinel LN biopsy Serum LDH (inc if metastasis) B-RAF mutation analysis CDKN2A gene testing ```
34
What is molluscum contagiosum?
A condition caused by the virus molluscum contagiosum, a ubiquitous pox virus that evades the immune system for many month-years. Causes a pearly papule with a central dell Lesions are generally caused via skin-to-skin or fomite contact in children and by sexual transmission in adults
35
What are the RFs?
Contact with infected, sex with infected, HIV, swimming, atopic dermatitis
36
What are the signs / symptoms of molluscum contangiosum?
``` Pearly papule with central dell, Possible erythema, possible atopic dermatitis Facial/groin disturbance Pruritus Sleep disturbance due to pruritus ```
37
What are the investigations for M contangiosum?
CLINICAL DIAGNOSIS - pearly white papules with central dell Can do curettage biopsy but rarely used HIV test Tzank staining - purple ovoid keratinocytes Haemotoxylin/eosin staining - Hederson - patterson bodies
38
What is a pressure sore?
Localised injury to the skin +/or underlying tissue as a result of sustained pressure, mainly on bony prominences.
39
What are the RFs for pressure sore formation?
Sensory deficit, inc age, inadequate blood flow, prolonged rest, ICU/surgery, poor mattress, environment eg wheelchair, DM, hx, malnourishment, incontinence
40
What are the 4 classifications + relevant signs for Pressure sores?
1 - localised skin injury - possible erythema (blanching), skin discolouration, painful, firm, mushy, boggy 2 - skin breakage - blisters/shiny indurated ulceration 3 - full thickness wound w/o bony involvement - some slough 4 - full thickness wound with slough / eschar and evidence of underlying involvement. Consider osteomyelitis, increased exudate and FOUL smell
41
What are the investigations needed for pressure sores?
CLINICAL DIAGNOSIS YO USUALLY NONE NEEDED Wound swab and culture ESR and WCC inc in infection Deep tissue biopsy/MRI if extensive
42
What is psoriasis?
A chronic inflammatory condition characterised by erythematous, scaly, flakey papules on the elbows, knees and extensor surfaces. Itchy BURNY STINGEHHH
43
RFs for psoriasis?
Genetic - IL23r IL12b TNFa INFECTION - post strep pharhygitis (URT INFECTIONS) Trauma = KOEBNERS phenomenon = near injection site/scars Stress/smoking/tight skin/ALCOOOOOL
44
What are the signs of psoriasis and 4 main subtypes?
Erythematous, scaly, flaky, itchy, circumscribed ``` Plaque = inflamed appearance with scaly eruptions Erythrodermic = generalised erythema with fine fine scaling Pustular = pus filled medical emergency Guttate = widespread erythematous, fine, scaly papules after URT INFECTION ```
45
What are the investigations for psoriasis?
CLINICAL Skin biopsy
46
SCC - what is it?
Proliferation of atypical transformed keratinocytes in the skin with malignant behaviour.
47
What are the RFs SCC?
Fits 1/2, fair skinned, actinic keratosis (precursor lesions), UV, Ionising radiation, sun exposure, latitude, skin phototype, prev cancer, tattoos, inc age, xeroderma pigmentosum
48
What are the signs and symptomsSCC?
Growing tumour, bleeding, non healing tender wound, dome shaped nodule with crater full of keratin in the centre, exophytic fungating plaques, ulceration, lymphadenopathy
49
What are the diagnostic tests for SCC?
Biopsy - evidence of atypical keratinocytes CT MRi PET - mets/LNs LFTs/CXR - mets
50
What is a sebaceous cyst?
The term sebaceous cyst is used to describe two types of cyst, both of which do not contain sebum. Pilar - arises from hair follicles Epidermoid - arises from the epidermis BOTH CONTAIN KERATIN
51
RFs for Seb cysts?
Testosterone, blocked glands, steroid use, Gardners syndrome / basal cell naevus syndrome
52
The S + S of SEB CYSTs
Don't occur on palms or soles Mobile masses containing fibrous tissue/fluids/purulent fluid/keratinous substance - contents differ due to infection PILAR - smooth mobile and painless Epidermoid - Pus, may hurt, usually in places with dec body hair
53
What are the investigations for seb cysts?
CLINICAL | USS to assess contents if atypical
54
Define acne rosacea
Rosacea is a common chronic disorder of the skin characterised by redness, flushing, and other cutaneous findings that often include telangiectases, roughened skin, rhinophyma, and general inflammation that can resemble acne. It primarily affects the convexities of the central face, including the cheeks, chin, nose, and central forehead, but it may extend to other parts of the body (usually the upper trunk). People with rosacea can also experience watery or irritated eyes. Overall, it is characterised by episodes of remission and recurrence. The diagnosis is usually clinical; avoidance of triggers is key to management. The most common manifestations are flushing, dilated prominent telangiectases (primarily on the face), persistent facial erythema, inflammatory papules, and pustules on the peri-orificial face. Prominence of sebaceous glands on the nose may result in fibrosis and rhinophyma. Typically, mild disease is initially treated with topical metronidazole or oral tetracyclines. Other antibiotics and anti-inflammatory agents can be used as adjuncts.
55
Epidemiology of acne rosacea
Although rosacea can occur in all racial and ethnic groups, fair-skinned people of Celtic and northern European heritage are most affected. Rosacea is rare in black people. Women are affected more often than men, and the condition usually starts between the ages of 30 and 50. Rosacea is estimated to affect over 14 million people in the US (around 5% of the population); outside the US, rosacea affects white people with an incidence similar to that in the US.
56
RFs for acne rosacea
``` STRONG Fair skin Hot baths/showers Temperature extremes Sunlight Emotional stress Hot drinks ``` WEAK Spicy foods Alcohol Medicines
57
Sx of acne rosacea
``` COMMON Flushing Erythema Papules + pustules Telangiectasias ``` Ocular manifestations - Other ocular manifestations include recurrent chalazion or hordeolum, keratitis, episcleritis, and scleritis. It is not uncommon for people to present with signs and symptoms suggestive of conjunctivitis and blepharitis, and for these to be significant manifestations of rosacea. Plaque formation Facial distribution - Rosacea primarily affects the convexities of the central face, including the cheeks, chin, nose, and central forehead. Concomitant acne vulgaris ``` UNCOMMON Phymatous change - The most frequent is rhinophyma (commonly known as 'whiskey nose' or 'rum blossom'). Burning / stinging Dry appearance Oedema Peripheral location ```
58
Ix for acne rosacea
Clinical Skin biopsy - May be performed to exclude more ominous conditions that mimic rosacea, including sarcoidosis, connective tissue disease, and mastocytosis. ANA - Perform if systemic lupus erythematosus (SLE) or other connective tissue disease is suspected.
59
Rx of acne rosacea
Topical ABx - metronidazole Oral ABx - doxycycline / tetracycline / clarithromycin Generally try metro + doxy first +/- benzoyl peroxide Laser therapy - For immediate relief of telangiectases and some background erythema (persistent flushing/vasodilation) SEVERE - can try isotretinoin Ocular manifestations - artificial tears and cleaning of the lids with warm water twice daily.
60
Prognosis of acne rosacea
The long-term outlook and prognosis varies considerably: Many people are unaware of their condition and would not opt for any one specific treatment even if given the choice. People with mild forms of rosacea may be able to keep their symptoms under control simply by avoiding vasodilating triggers and irritants. Many people with rosacea are quite satisfied with their response to treatment, having only mild symptoms, possibly with sporadic flushing only, and will not require any regular medicines. Other patients find no appreciable improvement, despite extensive efforts at quelling the symptoms using a great variety of treatment modalities.
61
A 45-year-old woman presents with several years' history of easy blushing, facial redness, and small, prominent blood vessels on the face. On physical examination, there are numerous telangiectases on a background of blanching erythema. Closer inspection demonstrates erythematous papules and pustules mostly on the central peri-orificial face.
acne rosacea Other presentations Atypical presentations include people in their 20s or 30s, children, and people with only ocular findings.
62
Aetiology of acne rosacea
The underlying cause is currently unknown. Nevertheless, factors or triggers that are known to precipitate the onset include: ``` Climatic exposures Vasculature Chemical and ingested agents Inflammation. There appears to be an exaggerated vasodilatory response to increased temperature, and redness can be easily exacerbated by hot drinks and hot baths or showers. The most important climatic exposure is sunlight. ``` General inflammation can be induced by chemical agents such as medicines (e.g., amiodarone), spicy foods, nasal corticosteroids, and, paradoxically, topical corticosteroids.
63
Define acne vulgaris
Acne vulgaris is a skin disease affecting the pilosebaceous unit. It is characterised by comedones, papules, pustules, nodules, cysts, and/or scarring, primarily on the face and trunk. Clinical manifestations range from mild comedonal acne to severe nodulocystic acne, which can be permanently disfiguring. In addition to the physical lesions, acne can have profound psychological and social impact on patients. Acne may affect any age group, but it is most common in adolescents. Lesions consist of non-inflammatory comedones (whiteheads and blackheads) and inflammatory papules, pustules, nodules, and cysts. Systemic effects may be present with acne fulminans, a rare variant of nodulocystic acne. Treatments include topical retinoids, keratolytics, and antibiotics; severe nodulocystic acne may require oral isotretinoin.
64
Epidemiology of acne vulgaris
Prevalence varies greatly worldwide, with higher rates of acne in developed countries compared with developing countries (some of which report no acne at all) Research in the US has shown that 85% of people between the ages of 12 and 24 years have acne, and while it is most common in teenagers, acne affects 8% of adults aged 25 to 34 years and 3% of adults aged 35 to 44 years. Acne in young adults may represent continuation of adolescent acne or development of late-onset disease Acne is more common in boys than girls during adolescence, but the incidence is higher in women during adulthood. Nodulocystic acne is more prevalent in white people compared with black people.
65
Aetiology of acne vulgaris
Acne is polygenic and multi-factorial. Four main pathogenetic factors contribute to the condition: Sebaceous gland hyperplasia and excess sebum production. Sebaceous follicle size and number of lobules per gland are increased in patients with acne.[10] Androgens stimulate sebaceous glands to enlarge and produce more sebum, which is most prevalent during puberty. Abnormal follicular differentiation. In normal follicles, keratinocytes are shed as single cells into the lumen and then excreted. In acne, keratinocytes are retained and accumulate due to their increased cohesiveness.[11] Cutibacterium (Propionibacterium) acnes colonisation. These gram-positive, non-motile rods are found deep in follicles and stimulate the production of pro-inflammatory mediators and lipases. While there may be increased numbers of C acnes in acne, bacterial counts often do not correlate with acne severity.[12] Inflammation and immune response. Inflammatory cells and mediators efflux into the disrupted follicle, leading to the development of papules, pustules, nodules, and cysts. External factors occasionally contribute to acne, including mechanical trauma, cosmetics, topical corticosteroids, and oral medicines (corticosteroids, lithium, iodides, some antiepileptic drugs). Endocrine disorders resulting in hyperandrogenism may also predispose patients to developing acne. Acne fulminans is a rare acne subtype that presents with variable systemic manifestations, including fever, arthralgias, myalgias, hepatosplenomegaly, and osteolytic bone lesions.
66
RFs for acne vulgaris
``` STRONG 12-24yo Genetic predisposition Greasy skin/increased sebum Medications ``` ``` WEAK Endocrine disorders Dietary factors Female sex/oestrogens Obesity/insulin resistence Hyperandrogenism Halogenated aromatic hydrocarbons exposure ```
67
Sx of acne vulgaris
Open and closed comedones represent the non-inflammatory acne lesions, while papules, pustules, nodules, and cysts are manifestations of inflammatory lesions. Post-inflammatory hyperpigmentation and scarring may also result. Tenderness Depression + anxiety Acne fulminans subtype manifests with fever, arthralgias, myalgias, hepatosplenomegaly, and osteolytic bone lesions.
68
Ix for acne vulgaris
Clinical Consider hormonal evaluation for hyperandrogenism - elevated total testosterone, dehydroepiandrostenedione sulphate (DHEA-S), luteinising hormone (LH), follicle stimulating hormone (FSH)
69
Rx of acne vulgaris
1. Topical retinoid or ABx eg tretinoin topical or clinda/erythro 2. Benzoyl peroxide OR Azelaic acid 3. Oral ABx 4. Oral retinoid - isotretinoin Corticosteroids may be used as an adjunct to isotretinoin or to calm skin down before initiating isotretinoin, typically for 1 to 4 months to avoid relapses. IF FEMALE can consider COC
70
Prognosis of acne vulgaris
Acne typically improves as people progress through adolescence, but it may persist into adulthood. Most people do not have long-term consequences from acne, but severe lesions may leave residual scarring in previously involved areas. Fine peeling and dryness caused by topical treatments tend to improve with continued use.
71
Complications of acne vulgaris
Scarring Dyspigmentation Depression
72
A teenage boy presents with closed comedones and slightly tender erythematous papules and pustules on his forehead, cheeks, chin, chest, and upper back. Small lesions developed a few years ago as he entered puberty, and they have progressively worsened over the last year. Previous lesions have left residual red-brown hyperpigmentation.
acne vulgaris Other presentations Acne conglobata is severe, nodulocystic acne without associated systemic manifestations.[4] Acne fulminans is the most severe form of cystic acne, characterised by the acute onset of nodular and suppurative acne associated with multiple systemic manifestations, including fever, arthralgias, myalgias, hepatosplenomegaly, and osteolytic bone lesions.[4] Acne mechanica is caused by persistent mechanical obstruction, often by clothing or sporting equipment, which occludes the pilosebaceous unit and results in comedones.[5] Acne excoriee des jeunes filles typically occurs in girls who scratch and pick at comedones and inflammatory papules. Acneiform eruptions can be caused by some medicines (e.g., androgens, topical corticosteroids, oral corticosteroids) and occupational or environmental exposures (e.g., chloracne caused by dioxins).
73
Rx of BCC
The National Comprehensive Cancer Network in the US recommends that for well-circumscribed BCC lesions less than 2 cm in diameter, excision with 4 mm clinical margins should result in complete removal in more than 95% of cases. Curettage and cautery by a dermatologist is another excellent approach. Although radiotherapy shows lower recurrence rates, comparable to the best surgical treatments, significant complications are possible (e.g., permanent hair loss, radiation burn, increased risk of secondary cancers). An example of where radiotherapy would be appropriate is for an elderly person with BCC on the nasolabial fold. Imiquimod: has been shown to be successful in 90% of the patients who are able to follow this therapy for at least 1 month; consists of daily or every-other-day application of topical imiquimod; patients should be aware that in about 10% of cases a surgical treatment might still be needed. Fluorouracil: topical application is helpful in management of multiple BCCs found on the head and neck, trunk, and legs. CRYOSURGERY METASTATIC DISEASE -> Vismodegib + Mohs Surgery - During the surgery, after each removal of tissue and while the patient waits, the tissue is examined for cancer cells. That examination informs the decision for additional tissue removal.
74
Prognosis of BCC
Mohs surgery After Mohs surgery, recurrence rates are <1% Surgical treatment After surgical treatment a scar will be visible, but recurrences are seen in <2%. Immunotherapy Immunotherapy has higher recurrence rates, but no scars. High-risk features Some features of BCC are indicative of aggressive tumour behaviour and are high-risk for recurrence: Tumour size 2 cm or larger Tumour site (the central face) Poorly defined clinical margins High-risk histological sub-type Histological features of aggression; perineural or perivascular involvement Failure of previous treatment (the tumour is a recurrence) Immunosuppression
75
Complications of BCC
LN spread Distant mets Local spread
76
A 63-year-old man with sun-damaged skin presents with a small nodule on the left aspect of his forehead. He mentions that it is itchy at times, and he thinks that he may have seen a colleague 2 years previously for removal of some keratoses or scabs. The patient indicates that these were either cauterised or frozen. On examination there is a pearly white nodule with prominent telangiectasia on its surface.
BCC Rarely, a patient might present with an advanced BCC, several centimetres in diameter and either deeply eroding into the surrounding tissue or with metastatic disease to lymph nodes and/or distant organs. Metastases are uncommon but have been described in the lungs and bones.
77
A woman in her mid-40s with dark, leathery skin and intense wrinkling of the lower neck (signifying excessive sun exposure either in a form of frequent sun tanning beds or perhaps frequent beach visits) presents at your office. She reports she has had multiple facial lifts, to decrease wrinkles. The plastic surgeon she has consulted performed other cosmetic procedures, including botulinum toxin type A injections. She complains about a mole on her jaw that has recently started to bleed.
BCC Rarely, a patient might present with an advanced BCC, several centimetres in diameter and either deeply eroding into the surrounding tissue or with metastatic disease to lymph nodes and/or distant organs. Metastases are uncommon but have been described in the lungs and bones.
78
Define dermatofibroma
A dermatofibroma is a common benign fibrous nodule that most often arises on the skin of the lower legs. A dermatofibroma is also called a cutaneous fibrous histiocytoma.
79
Epidemiology of dermatofibroma
Dermatofibromas occur at all ages and in people of every ethnicity. They are more common in women than in men.
80
RFs for dermatofibroma
It is not clear if dermatofibroma is a reactive process or if it is a neoplasm. The lesions are made up of proliferating fibroblasts. Histiocytes may also be involved. They are sometimes attributed to an insect bite or rose thorn injury, but not consistently. They may be more numerous in patients with altered immunity.
81
Sx of dermatofibroma
Dermatofibromas most often occur on the legs and arms, but may also arise on the trunk or any site of the body People may have 1 or up to 15 lesions. Size varies from 0.5–1.5 cm diameter; most lesions are 7–10 mm diameter. They are firm nodules tethered to the skin surface and mobile over subcutaneous tissue. The skin dimples on pinching the lesion. Colour may be pink to light brown in white skin, and dark brown to black in dark skin; some appear paler in the centre. They do not usually cause symptoms, but they are sometimes painful or itchy.
82
Ix for dermatofibroma
Dermatofibroma is usually easy to diagnose clinically, supported by dermatoscopy. The most common dermatoscopic pattern is a central white area surrounded by a faint pigment network. Diagnostic excision or skin biopsy is undertaken if there is an atypical feature such as recent enlargement, ulceration, or asymmetrical structures and colours on dermatoscopy. The pathology of dermatofibroma shows whirling fascicles of spindle cell proliferation with excessive collagen deposition in the dermis. There are several pathological variants. ``` cellular aneurysmal epithelioid atypical lipidized ankle-type palisading cholesterotic In case of doubt, immunohistochemical staining is used to confirm the diagnosis. ```
83
Rx of dermatofibroma
A dermatofibroma is harmless and seldom causes any symptoms. Usually, only reassurance is needed. If it is nuisance or causing concern, the lesion can be removed surgically. Cryotherapy, shave biopsy and laser treatments are rarely completely successful.
84
Prognosis / complications of dermatofibroma
Because they are often raised lesions, they may be traumatised, for example by a razor. Occasionally dozens may erupt within a few months, usually in the setting of immunosuppression (for example autoimmune disease, cancer or certain medications). Dermatofibroma does not give rise to cancer. However, occasionally, it may be mistaken for dermatofibrosarcoma or desmoplastic melanoma.
85
Define seborrheic keratosis
Seborrhoeic keratosis is a common, multiple, benign skin tumour most commonly found on the torso and usually appears in the fourth and fifth decades of life. From 80% to 100% of people over the age of 50 years are affected. The lesions clinically appear as well-circumscribed grey-brown-to-black plaques with a 'stuck-on' appearance. Later the plaques can become raised and may show a verrucous surface. Most lesions do not exceed 1 cm in diameter. The lesions are normally painless and require no treatment Lesions are common, multiple, benign tumours of the skin. Most people over the age of 50 years are affected. They appear as well-circumscribed 'stuck-on' plaques or papules and may look like warts. They are usually asymptomatic but can become irritated and inflamed spontaneously or because of friction from clothing. Treatment is not necessary because of their benign nature, but if irritated, itching, and displeasing, can be initially treated with cryotherapy and curettage. Most important differential diagnosis, and of patient concern, is malignant melanoma. Can be differentiated from melanoma by the SURFACE features. Seborrheic keratosis will have smooth surfaces.
86
Epidemiology of seborrheic keratosis
Seborrhoeic keratosis is one of the most common skin tumours. From 80% to 100% of people over the age of 50 years present with seborrhoeic keratosis. It is uncommon in people under 30 years of age. Men and women are affected equally.
87
Aetiology of seborrheic keratosis
Familial predisposition is suspected, with an autosomal-dominant inheritance. They are found in 80% to 100% of people over 50 years old. There is a possible association with chronic UV damage.
88
RFs for seborrheic keratosis
``` STRONG >50 White FHx Sun / UV ``` WEAK Black people - dermatosis papulosa nigra (Dermatosis papulosa nigra describes the presence of multiple, small, 1–5 mm diameter, smooth, firm, black or dark brown papules on face and neck.- almost like moles) Female - dermatosis papulosa nigra Pregnancy
89
Sx of seborrheic keratosis
``` Stuck on lesions Localisation to torso/face Yellow-brown colour Slightly raised FLAT surface Wart like texture Multiple lesions Painless Itching Horn pearls on surface (yellow white areas) ``` Can be differentiated from melanoma by the SURFACE features. Seborrheic keratosis will have smooth surfaces.
90
Ix for seborrheic keratosis
Dermoscopy - The characteristic dermoscopic features of seborrhoeic keratosis are milia-like cysts and comedo-like openings. Milia-like cysts are white or yellowish round structures that correspond to small intra-epidermal, keratin-filled cysts (horn pearls). They are not exclusively found in seborrhoeic keratosis as they are also seen in congenital nevi and papillomatous dermal nevi, but less frequently. Comedo-like openings (also called pseudofollicular openings or crypts) are brownish holes in the surface of seborrhoeic keratosis that correspond histologically to keratin-filled invaginations of the epidermis. Biopsy Reflectance confocal microscopy
91
Rx of seborrheic keratosis
Itchiness -> corticosteroids betamethasone dipropionate topical: (0.05%) apply sparingly to the affected area(s) once or twice daily for 4-5 days RAISED -> Curettage leaves a flat surface that becomes covered by normal epidermis in 1 week. Cautery is used scarcely as it is more likely to leave scars. Other complications of both curettage and cautery are hyperpigmentation, hypopigmentation, and keloids. FLAT -> cryotherapy / curettage / laser / tretinoin
92
Prognosis of seborrheic keratosis
Patients who elect for treatment are likely to see good results.
93
Complications of seborrheic keratosis
Irritation and itching | Misdiagnosis
94
An otherwise healthy 60-year-old man presents with multiple raised pigmented lesions over his back and chest. He complained that these lesions were gradually increasing in size and number. The patient is worried about melanoma. He remembered that his parents and other family members had similar lesions. He commented that some spots are extremely pruritic. The examination of the patient's skin revealed multiple light- and dark-brown warty papules and plaques on his torso with a 'stuck-on' appearance.
seborrheic keratosis
95
Define vitiligo
Vitiligo is an acquired loss of melanocytes in circumscribed areas of the epidermis, resulting in complete depigmentation of affected skin. The extent of the disease ranges from limited, focal disease to almost complete (universal) pigment loss. Common acquired multi-factorial skin disease exhibiting progressive depigmentation of the epidermis in circumscribed areas, typically without erythema or scaling. Multiple auto-immune disorders may be associated with vitiligo, most commonly auto-immune thyroiditis. In light-skinned people and in sun-protected areas, Wood's lamp examination helps with diagnosis and evaluation of extent of disease. Presents typically as round, depigmented macules and patches that slowly enlarge, most often in a symmetrical distribution, and with a particular affinity to periorificial and acral skin. Hair in affected areas may be white, particularly in areas of long-standing disease. When repigmentation occurs, it typically occurs in a perifollicular pattern. While there are no licensed treatments to induce repigmentation, treatments such as UV-B light, topical immunosuppressants, and combination approaches using phototherapy can provide significant improvement in most cases.
96
Epidemiology of vitiligo
Vitiligo is a common disorder affecting all races and geographic areas with a prevalence of 0.5% to 2% worldwide, and approximately 0.5% in the US There is an increased prevalence in people with a FHx of the condition. Vitiligo may manifest at any time in an individual's lifespan with an average age at onset of 20 years. Men and women are equally affected. A minority of cases (10% to 15%) in adults present in the segmental form. In children, the segmental variant is more prevalent. There is an increased occurrence with exposure to chemicals that are toxic to melanocytes
97
RFs for vitiligo
``` STRONG Age <30 FHx AI disease Chemical contact ``` The disease most often associated with vitiligo is auto-immune thyroid disease. Less commonly associated conditions include pernicious anaemia, SLE, alopecia areata, type 1 diabetes, and Addison's disease
98
Sx of vitiligo
``` COMMON Acral and periorificial depigmentation Perianal and genital depigmentation Recent cutaneous trauma Localised sunburn pain Enhancement + fluorescence with UVA exposure Halo Naevus ``` ``` UNCOMMON Universal depigmentation Lip depigmentation Ertyhema or blistering Retinal depigmentation ```
99
Ix for vitiligo
Clinical Skin biopsy possible
100
Rx of vitiligo
Topical corticosteroid and or tacrolimus clobetasol topical: (0.05%) apply to the affected area(s) twice daily, maximum 50 g/week OR mometasone topical: (0.1%) apply to the affected area(s) once daily OR tacrolimus topical: (0.1%) apply to the affected area(s) twice daily CONSERVATIVE: Patients should be advised to avoid cutaneous trauma where possible (e.g., physical trauma, surgical incisions, friction, sunburn) Targeted phototherapy devices (excimer laser or monochromatic lamp) that deliver light in the UV-B range (peak at 308 nm) can be considered as second-line treatment in patients who do not respond to topical therapies Surgery can be considered if medical treatment is unsatisfactory to the patient and disease is stable. Several techniques are available to achieve this, including punch grafting, epidermal blister grafting, or ultrathin epidermal sheet grafting.
101
Prognosis of vitiligo
Vitiligo is a progressive disease that may arrest after an initial phase of variable duration. Phases of partial repigmentation, depigmentation, or progression may follow. Complete repigmentation without treatment is extremely rare. Localised forms of vitiligo may remain stable and restricted to a limited area, but may also spread to become generalised. About one third of vitiligo patients experience partial repigmentation in sun-exposed patches. This finding may predict a good treatment response. The majority of patients experience greater than 75% repigmentation when following first-line therapies. A general rule of thumb is to expect 25% improvement after 3 months, 50% improvement after 6 months, and 75% improvement after 9 months of therapy. Long-standing disease, a segmental distribution, and poliosis are considered predictors of a less-favourable treatment outcome. Although 50% repigmentation rates in affected skin are highly significant in statistical terms, they may not be satisfactory to the patient. Notably, glabrous skin such as the penis, hands, wrists, feet, and fingertips often do not respond at all
102
Complications of vitiligo
Up to 50% of patients report Koebner's phenomenon (i.e., depigmentation in response to trauma) ``` Photosensitivity Rx induced permanent photosensitivity Dermatoheliosis Skin cancer Psychologica; Retinal + choriod depigmentation Audiological compromise ```
103
A 21-year-old Hispanic woman presents with localised sunburn around her eyes. The affected skin is notable for pigment loss with additional white macules appearing around the fingertips and on the elbows. On PE there are well-demarcated and depigmented patches of skin, without scaling or infiltration, in a generalised distribution over the body. Small patches are noted on the perineum. Additional affected areas were identified on both hips with Wood's lamp examination. A depigmented halo surrounds 1 naevus on the right scapula. Patches in sun-exposed areas show small, perifollicular macules of pigmentation.
vitiligo Other presentations of vitiligo include inflammatory vitiligo (inflammatory infiltrate at the tissue level reflected in a slightly raised erythematous border, often with scale, of actively enlarging lesions), trichrome vitiligo (a third colour, a shade between depigmented and normally pigmented skin, is present), and confetti-like depigmentation (many tiny 1-2 mm macules of depigmentation in clusters). These presentations frequently mark highly active disease.
104
Aetiology of vitiligo
Vitiligo is caused by the T-cell-mediated destruction of melanocytes. Intrinsic abnormalities present in melanocytes probably initiate inflammation through the activation of innate immunity. This inflammation leads to the recruitment of T cells, including cytotoxic T cells that are melanocyte-specific, recognising antigens produced specifically by melanocytes. These cytotoxic T cells destroy the melanocytes, therefore leading to the loss of melanin production and pigmentation of the overlying keratinocytes.
105
Define alopecia areata
Alopecia areata (AA) is an autoimmune disease that affects almost 2% of the population in the US. Inflammatory cells target the hair follicle, thus preventing hair growth. Typically, a small round patch of hair is noticed. This patchy hair loss may regrow spontaneously. In other cases there can be extensive patchy hair loss and in rare cases there is loss of all scalp and body hair (alopecia areata universalis). Autoimmune disease that targets the hair follicle. Characterised by sudden onset of patchy hair loss. Diagnosis is clinical. Characteristic physical findings are exclamation mark hairs (short, broken hairs) and a positive pull test. A scalp biopsy shows perifollicular inflammation. Topical or intralesional corticosteroids are appropriate for patients with limited hair loss. Skin atrophy can be a temporary side effect.
106
Epidemiology of alopecia areata
The most reliable data come from a US population-based study, which estimated that AA affects almost 2% of the US population. The study showed that it affects both sexes equally and is seen in all age groups with no known ethnic predominance. A higher incidence is found in children and young adults. Patients usually present with several episodes of hair loss and hair regrowth during their lifetime. The course is irregular and unpredictable. The recovery from hair loss may be complete, partial, or none. The incidence of the severe chronic form of the condition is 7% to 10% in affected people Indicators of a poor prognosis are atopy, the presence of other immune diseases, family history, young age at onset, nail dystrophy, extensive hair loss, and ophiasis (hair loss at the inferior hair line of the scalp [i.e., inferior-parietal/occipital scalp])
107
RFs of alopecia areata
WEAK AI FHx
108
Aetiology of alopecia areata
The aetiology of AA has not been determined. However, it is hypothesised to be an organ-specific autoimmune disease mediated by T lymphocytes directed at hair follicles. With respect to the genetics of AA, there seem to exist both susceptibility and severity genes
109
Sx of alopecia areata
Hair loss Non-scarring alopecia Exclamation mark hairs - Hairs that are more narrow or attenuated at the proximal end (near the skin) and may be blunt or broken at the distal end. Positive hair pull test - A pinch of closely grouped hairs (or about 50 hairs) is grasped between the thumb and the index and middle fingers, and gentle traction is applied as the fingers are pulled firmly and slowly away from the scalp. In normal adults, 2 to 5 hairs will be obtained in this manner. More than 5 hairs is considered to be a positive pull test. Nail pitting
110
Ix for alopecia areata
Clinical Consider dermoscopy / biopsy
111
Rx of alopecia areata
1. Topical corticosteroid clobetasol topical: (0.05%) apply sparingly to the affected area(s) twice daily + cosmetic camouflage + patient support 2. Topical minoxidil minoxidil topical: (2% or 5% solution) apply 1 mL to the affected area(s) twice daily; (5% foam) apply half a capful to the affected area(s) twice daily 3. Intralesional corticosteroid Intralesional corticosteroids are appropriate initial treatment for adults with localised disease (limited areas of patchy hair loss). triamcinolone acetonide: 2.5 to 5 mg intralesionally to brows, beard, or scalp every 4-6 weeks until full regrowth 4. ORAL corticosteroid prednisolone: 1 mg/kg/day orally for two weeks, then taper dose gradually over the next four weeks for a total of six weeks treatment _______________________ Patients who have extensive hair loss or who do not respond well to the above measures should be referred to a specialist for further treatment. Topical immunotherapy may be tried with either DNCB (1-chloro-2,4-dinitrobenzene), DPCP (diphenylcyclopropenone), or SADBE (squaric acid dibutyl ester).
112
Prognosis of alopecia areata
The natural history of alopecia areata (AA) is that of unpredictable episodes of relapsing and remitting disease. Within 2 years, 90% of the patients with limited scalp involvement will have spontaneous regrowth. These patients with limited scalp involvement are also the ones who are more likely to respond to any form of treatment. The presence of alopecia totalis/universalis for >2 years, severe nail abnormalities, atopy, and onset in childhood (<5 years) are factors thought to be associated with a poor response to treatment. In addition, a long duration and early onset of disease are indicators of poor prognosis.
113
Complications of alopecia areata
topical minoxidil-induced skin irritation and hypertrichosis intralesional corticosteroid-induced adverse effects oral corticosteroid-induced adverse effects topical immunotherapy-induced sensitisation topical corticosteroid-induced adverse effects psychological distress topical immunotherapy-induced dyspigmentation
114
A 20-year-old woman presents with a 1-month history of patchy hair loss. The patient denies any symptoms of scalp itching or irritation. Past medical history is significant for mild asthma and eczema. Her aunt has rheumatoid arthritis. On physical examination, she has a circular area of hair loss measuring approximately 25 mm in diameter in the occipital scalp. The scalp in that area is slightly salmon-coloured. The follicular orifices are intact and there is evidence of exclamation mark hairs (short, broken hairs). A hair-pull test is positive at the periphery of the alopecia patches (a sign of activity). Her brows and lashes appear normal, but her fingernails have evidence of fine ridges.
alopecia areata Other presentations Loss of facial and/or body hair (including brows, lashes, or beard) is less common. Less common presentations are complete scalp hair loss with (alopecia universalis) or without (alopecia totalis) body hair loss. Diffuse hair loss is a rare presentation of alopecia areata.
115
Define dermatophyte infections / tinea
Superficial fungal infection with varying presentation depending on site. Dermatophytes are fungal organisms that require keratin for growth. These fungi can cause superficial infections of the hair, skin, and nails. Dermatophytes are spread by direct contact from other people, animals, soil, and from fomites. Diagnosis is typically clinical, although speciation via fungal culture and proof of mycological cure via serial fungal culture may aid patient care. Confirm diagnosis of tinea unguium and tinea capitis prior to treatment. Topical therapy is sufficient for most tinea infections. Fungicidal topical allylamines demonstrate good cure rates with short duration of treatment. Systemic therapy is preferred for tinea capitis, tinea barbae, tinea manuum, and onychomycosis.
116
Epidemiology of dermatophyte infections / tinea
Estimates of the incidence and prevalence of dermatophyte infections vary. Tinea pedis: the most common of the superficial fungal infections, occurring in up to 70% of adults.[1] Onychomycosis: the most prevalent condition affecting nails, accounting for approximately 50% of all nail disease.[2] One study reported 2% prevalence of onychomycosis in the paediatric-age population, increasing to 60% in those aged 80 years and over.[3] Tinea capitis: mainly affects pre-adolescent children, with one study reporting peak incidence in black boys under the age of 10.[4] Tinea corporis: common, with highest prevalence in pre-adolescents in hot, humid climates. Tinea cruris: most prevalent in adolescent and adult men. Tinea barbae: uncommon and most likely to be found in men who have been in direct contact with infected farm animals. Majocchi's granuloma = little leg/butt spots
117
Aetiology of dermatophyte infections / tinea
Host factors: Genetic susceptibility, including atopy Immunosuppressive illnesses or medications (e.g., corticosteroids) Presence of other skin diseases that disrupt the epidermis (e.g., atopic dermatitis) Other illnesses that predispose to skin infection, including diabetes mellitus and peripheral vascular disease. Local factors: Sweating Occlusion Occupational exposure High humidity (tropical or semi-tropical climates) Exposure to infected pets or farm animals, infected fomites, skin contact with the floors of public bathing facilities Contact sports such as wrestling.
118
RFs for dermatophyte infections / tinea
``` STRONG Exposure to infected people animals or soil Exposure to fomites -> hats combs + hairbrushes Chronic corticosteroid use HIV DM Occlusive clothing Hot humid weather Obesity Hyperhidrosis Frequenting Public bathing areas bare foot Deformities of feet Recurrent trauma to skin ``` ``` WEAK Wrestling Atopic dermatitis +ve FHx Peripheral vascular disease ```
119
Sx of dermatophyte infections / tinea
History of a skin lesion present for days to months (tinea corporis, tinea manuum, tinea faciale, tinea cruris, Majocchi's granuloma, tinea pedis), scalp or beard lesion for days to months (tinea capitis, tinea barbae), or changes in toenails or fingernails for weeks to years (tinea unguium). Infected skin may burn, itch, or be tender (especially the vesiculobullous form affecting the interdigital areas of the feet). Scalp lesions in tinea capitis may be irregular or well-demarcated. Typical in tinea capitis. Lesions of face, trunk, extremities, hands, and groin (tinea faciale, corporis, manuum, cruris) reveal a characteristic pattern of inflammation, termed an active border. The inflammatory response is characterised by a greater degree of redness and scaling at the edge of the lesion, with occasional blister formation. Central clearing of the lesion is often present and distinguishes dermatophytoses from other papulosquamous eruptions such as psoriasis or lichen planus, in which there is a uniform inflammatory response throughout the skin lesion. Erythematous, scaling rash with follicular pustules in beard or moustache - Typical of tinea barbae, which involves the skin and coarse hairs of the beard and moustache area; occurs in adult men and hirsute women. Diffusely dry palmar surface with hyperkeratosis - Tinea manuum is a fungal infection of one or, occasionally, both hands. It often occurs along with tinea pedis (one hand, two feet syndrome). Jock itch / Tinea cruris affects the proximal, medial thighs. It can extend to the abdomen and buttocks. The scrotum tends to be spared. Pustules and vesicles are present at the active edge of the infected area, along with maceration due to pruritus. Evaluate the feet as a source of the infection. The vesiculobullous form of tinea pedis, or athlete's foot. Fissuring, maceration, and scaling in the interdigital spaces of the fourth and fifth toes. Chronically scaly, hyperkeratotic plantar skin with erythema of the soles, heels, and sides of the feet Folliculitis with nodules - Majocchi's granuloma is a foreign-body granuloma caused by dermatophytes, resulting in folliculitis with papules and pustules. Granulomatous nodules are typically in a distinct location, often the lower two-thirds of the leg in women who shave their legs. Thickened nail with subungual hyperkeratosis, onycholysis, and white-yellow to brown discoloration Small, white speckled patches on the surface of the nail plate with crumbling nail Cervical and occipital lymphadenopathy may be prominent in tinea capitis Sx of PAD
120
Ix for dermatophyte infections / tinea
Clinical If in doubt - potassium hydroxide (KOH) microscopy aids in confirming the diagnosis when unclear clinically. Must be used to confirm nail infection before starting treatment. OR wood lamp/culture/dermoscopy
121
Rx of dermatophyte infections / tinea
Systemic anti fungal therapy for tinea capitis, barbae, manuum, majocchis granuloma griseofulvin: adults: 500-1000 mg/day orally given in 1-2 divided doses for 4-8 weeks OR terbinafine: adults: 250 mg orally once daily for 6 weeks OR itraconazole: adults: 200 mg orally once daily for 2-4 weeks Secondary options fluconazole: adults: 200 mg orally once daily for 2-4 weeks Topical anti fungal shampoos for capitis: ketoconazole topical: (1-2% shampoo) adults: apply to scalp twice weekly for 4 weeks, leave each application on scalp for 5 minutes then rinse Other topicals available - aluminium acetate
122
Prognosis of dermatophyte infections / tinea
Prognosis for treatment of most tinea infections is excellent. However, individuals who are immunocompromised may have persistent infection or occasionally develop invasive dermatophyte disease. Onychomycosis is the most difficult of the tinea infections to treat and has a treatment failure rate of 20% to 40%,[56] and a recurrence or re-infection rate of 20% to 50%,[57] which is high enough to warrant periodic re-evaluation every few months during and after completion of treatment. Consider baseline liver function tests and periodic monitoring during systemic therapy and in older people.[9] In the case of Trichophyton rubrum syndrome, assure patient compliance for the need of continuous therapy until the complete clearance of infection from all sites, and the use of a topical drug in a proper manner and quantity.[9]
123
A 15-year-old boy presents with a month-long history of itching and scaling in the groin region. Physical examination reveals hyperpigmentation of skin on both upper medial thighs and inguinal areas with scaling and secondary excoriation. There is sparing of the scrotum and penis; also noted is some maceration between the fourth and fifth toe of his right foot that causes burning and itching. He competes as a member of his school swimming team.
dermatophyte infections / tinea Tinea cruris A characteristic feature of dermatophyte infections is an inflammatory pattern at the edge of the skin lesion, with scaling and redness or, occasionally, blister formation. The lesions can be located, in addition to the scalp, on the trunk, face, palmar area of the hands, groin (but sparing the scrotum and penis), and the fingernails and toenails.
124
Define pityriasis versicolor
Pityriasis versicolor (PV), also known as tinea versicolor, is a common superficial fungal infection of the stratum corneum (outer layer of the epidermis) resulting from a change to the mycelial (filamentous fungus or mould) state of dimorphic yeasts of the genus Malassezia. It leads to hypo- or hyperpigmented coalescing scaly macules on the trunk and upper arms. The eruption is most common in the summer months in adolescents and young adults, favours oily areas of the skin, and often has a relapsing nature requiring frequent treatment. A superficial fungal infection of the stratum corneum, due to dimorphic yeasts of the genus Malassezia, leading to hypo- or hyperpigmented macular lesions on seborrhoeic areas of the trunk. Eruption is most common in the summer months in adolescents. Often has a relapsing nature requiring frequent treatment or prophylaxis. Primarily a clinical diagnosis that is confirmed by a KOH preparation demonstrating fungal elements with a characteristic spaghetti-and-meatballs appearance indicating the presence of both yeast and short hyphae. Easily treated with either topical medications, including zinc pyrithione shampoo, selenium sulfide shampoo, or azole-class topical antifungal creams. More extensive disease may require systemic therapy with antifungal drugs. After successful treatment, patients should be reminded that it may take up to 6 weeks before their normal skin pigmentation returns. Recurrence of the disease is common, and prophylactic treatment with selenium shampoo and, for more extensive disease, systemic therapy with antifungal drugs may be necessary.
125
Epidemiology of pityriasis versicolor
The prevalence of PV is about 1% of the population in Scandinavia and 0.8% of the US population PV is most often seen in teenagers and young adults. It is unusual for young children and older adults to have PV due to reduced sebum production in these age groups
126
Aetiology of pityriasis versicolor
Hypopigmentation has been explained by Damage to melanocytes Inhibition of tyrosinase by decarboxylic acid[15] Lipoperoxidase[16] produced by Malassezia species Melanosomes of small size Decreased numbers of melanosomes in affected skin[17] Blocking of UV light by lipid-like material accumulating in the stratum corneum due to Malassezia species.[18]
127
RFs for pityriasis versicolor
``` STRONG High ambient temperature and humidity Adolescent and young adults Hyperhidrosis Systemic corticosteroid or other immunosuppressant use Athletes Greasy skin ``` ``` WEAK Fix Malnutrition Contraceptives HIV Occlusive ointments or creams ```
128
Sx of pityriasis versicolor
``` COMMON Lack or pruritus or pain Dyspigmentation Macules or patches Fine overlying scale Seborrheic distribution ``` UNCOMMON Yellow fluorescence under wood lamp
129
Ix for pityriasis versicolor
Clinical Culture + biopsy possible but seldom needed
130
Rx of pityriasis versicolor
Topical antifungals eg: ketoconazole topical: (2%) children and adults: apply to whole neck, trunk, arms, and legs once or twice daily for 2 weeks OR clotrimazole topical: (1%) children and adults: apply to whole neck, trunk, arms, and legs twice daily for 2 weeks OR miconazole topical: (2%) children and adults: apply to whole neck, trunk, arms, and legs twice daily for 2 weeks Topical retinoids (e.g., tretinoin, adapalene) may also be used. +/- UV therapy if individual is concerned re appearance (after eradication) 2. Systemic antifungals fluconazole: children: 3-6 mg/kg orally once weekly for 2 weeks; adults: 400 mg orally as a single dose, or 200 mg once weekly for 2 weeks OR itraconazole: adults: 400 mg orally once daily for 3 days, or 200 mg orally once daily for 7 days Secondary options ketoconazole: children >2 years of age: 6.6 mg/kg orally once weekly for 2 weeks, maximum 400 mg/dose; adults: 400 mg orally once weekly for 2 weeks
131
Prognosis of pityriasis versicolor
Spontaneous resolution of PV is uncommon, and the disease will persist for years if left untreated.[13] Fortunately, the treatment options for PV are safe and efficacious. However, even after successful treatment, patients and clinicians must remember that the pigmentary abnormalities associated with PV may take up to 6 weeks to resolve and that this is not a sign of treatment failure.[32] Hypopigmented lesions, in particular, can take longer to resolve.[19] In addition, because the conversion of Malassezia yeasts to the pathological mycelial form is thought to be due, in many cases, to endogenous host factors, recurrence is common in up to 60% of patients in the first year after treatment and up to 80% after 2 years.[6][33] Because of this, clinicians should consider prophylactic treatment for patients who tend to have repeated episodes of disease.[6]
132
Complications of pityriasis versicolor
DYSCHROMIA
133
A 20-year-old healthy white man on a basketball team presents with a several-month history of an asymptomatic eruption on the trunk and shoulders. He recalls having a similar eruption several years ago in the summer. On examination, there are slightly scaly, coalescing, fawn-coloured macules and patches on the neck, shoulders, and upper back and chest. There is no evidence of excoriation, erythema, induration, or central clearing.
pityriasis versicolor The lesions of PV vary in colour (hence versicolor) from hypopigmented and white (PV alba) to hyperpigmented shades of pink, salmon, brown, red (PV rubra) or, rarely, black (PV nigra).[3] PV is generally thought to cause hypopigmented lesions in dark skin and hyperpigmented lesions in light skin. However, one study could find no correlation between pigmentary variation in PV and age, sex, duration, symptoms, or background skin colour in darker skin.[4] PV may rarely be pruritic.[1] Commonly affected areas include the chest, back, shoulders, and neck.[3] Facial involvement is uncommon and is usually seen in tropical climates or in children in temperate climates.[3][5] Less-frequent sites of involvement include the scalp, legs, groin, or within a radiotherapy field.[1] PV is most common in the tropics and in the summer, and in adolescents and young adults.[2][6] Participation in athletics, pregnancy, hyperhidrosis, malnutrition, immunosuppression, use of oral contraceptives, and family history of PV are associated with the disease.[6]
134
Case history #2 A 45-year-old Hispanic woman with a medical history of renal failure necessitating renal transplant presents with asymptomatic areas on her shoulders, back, and chest that do not tan. Her current immunosuppressive regimen consists of prednisolone (prednisone) and azathioprine. On examination, she has scaly, hypopigmented, coalescing macules and patches on the neck, shoulders, chest, back, and abdomen. There is no evidence of excoriation, erythema, induration, or central clearing.
pityriasis versicolor The lesions of PV vary in colour (hence versicolor) from hypopigmented and white (PV alba) to hyperpigmented shades of pink, salmon, brown, red (PV rubra) or, rarely, black (PV nigra).[3] PV is generally thought to cause hypopigmented lesions in dark skin and hyperpigmented lesions in light skin. However, one study could find no correlation between pigmentary variation in PV and age, sex, duration, symptoms, or background skin colour in darker skin.[4] PV may rarely be pruritic.[1] Commonly affected areas include the chest, back, shoulders, and neck.[3] Facial involvement is uncommon and is usually seen in tropical climates or in children in temperate climates.[3][5] Less-frequent sites of involvement include the scalp, legs, groin, or within a radiotherapy field.[1] PV is most common in the tropics and in the summer, and in adolescents and young adults.[2][6] Participation in athletics, pregnancy, hyperhidrosis, malnutrition, immunosuppression, use of oral contraceptives, and family history of PV are associated with the disease.[6]
135
Case history #2 A 45-year-old Hispanic woman with a medical history of renal failure necessitating renal transplant presents with asymptomatic areas on her shoulders, back, and chest that do not tan. Her current immunosuppressive regimen consists of prednisolone (prednisone) and azathioprine. On examination, she has scaly, hypopigmented, coalescing macules and patches on the neck, shoulders, chest, back, and abdomen. There is no evidence of excoriation, erythema, induration, or central clearing.
pityriasis versicolor The lesions of PV vary in colour (hence versicolor) from hypopigmented and white (PV alba) to hyperpigmented shades of pink, salmon, brown, red (PV rubra) or, rarely, black (PV nigra).[3] PV is generally thought to cause hypopigmented lesions in dark skin and hyperpigmented lesions in light skin. However, one study could find no correlation between pigmentary variation in PV and age, sex, duration, symptoms, or background skin colour in darker skin.[4] PV may rarely be pruritic.[1] Commonly affected areas include the chest, back, shoulders, and neck.[3] Facial involvement is uncommon and is usually seen in tropical climates or in children in temperate climates.[3][5] Less-frequent sites of involvement include the scalp, legs, groin, or within a radiotherapy field.[1] PV is most common in the tropics and in the summer, and in adolescents and young adults.[2][6] Participation in athletics, pregnancy, hyperhidrosis, malnutrition, immunosuppression, use of oral contraceptives, and family history of PV are associated with the disease.[6]
136
Name the cancer associated with Acanthosis Nigricans Maligna
Abdominal adenocarcinomas (gastric neoplasia)
137
Name the cancer associated with Acquired Pachydermatoglyphia = tripe hands velvet hands / super wrinkled
Gastric and pulmonary carcinoma
138
Name the cancer associated with Erythema gyratum repens | crazy migrating erythema that looks like cool patterns
Pulmonary, esophageal and breast
139
Name the cancer associated with Bazex paraneoplastic acrocheratosis (hyperkeratotic lesions on the nose, ears, palms, and soles)
Aerodigestive tract (oral cavity, larynx, pharynx, trachea, esophagus and lung)
140
Name the cancer associated with Acquired hypertrichosis lanuginosa - (very rare condition characterised by the rapid growth of long, fine, lanugo-type hair particularly around the eyebrows, forehead, ears and nose in adulthood.)
Colorectal, pulmonary and breast
141
Name the cancer associated with Necrolytic migratory erythema (The rash fluctuates in severity. Initially there is a ring-shaped red area that blisters, erodes and crusts over. It can be quite itchy and painful. As it heals, it may leave behind a brown mark. It also results in a sore smooth tongue, a sore mouth, cracked dry lips and ridging of the nails.)
Glucagonoma
142
Name the cancer associated with Leser-Trélat Sign (abrupt appearance of multiple seborrhoeic keratoses that rapidly increase in their size and number)
Gastric and colorectal
143
Name the cancer associated with Paraneoplastic pemphigus
Non-Hodgkin lymphoma, chronic lymphocytic leukemia, Castleman's disease, thymoma
144
Name the cancer associated with Pityriasis rotunda (round or oval scaly, pigmented patches that mainly occur on the trunk, arms and legs. There appear to be two types of pityriasis rotunda.)
Hepatocellular carcinoma, gastric and esophageal carcinoma, prostate cancer, chronic lymphocytic leukemia and multiple myeloma
145
Name the cancer associated with Dermatomyositis
Ovarian carcinoma, bronchogenic adenocarcinoma
146
Name the cancer associated with Palmoplantar keratoderma ‘Keratoderma’ is a term that means marked thickening of the skin.
Esophageal Carcinoma
147
Name the cancer associated with Pyoderma gangrenosum
Myelodysplastic syndrome, myeloma, leukemia
148
Name the cancer associated with Sweet syndrome (Sweet’s syndrome (also known as acute febrile neutrophilic dermatosis) is a rare skin disorder characterised by a fever and the appearance of tender red or purple lumps or patches on the skin that may ulcerate.)
Acute myelogenous leukemia, myelodysplastic syndrome
149
Define scabies
Scabies is caused by infestation with the ectoparasite Sarcoptes scabiei, a mite that burrows through the human stratum corneum. Spread is primarily via direct contact with an individual with scabies. Clinical clues to diagnosis include intense pruritus and linear erythematous burrows, particularly on the extremities, or erythematous papules and nodules elsewhere such as in the axilla or genital area. Rarely, it can present on the neck and scalp. Diagnosis is based on history and clinical appearance Highly contagious via direct skin-to-skin contact; seen most commonly in overcrowded living conditions and in developing countries. Caused by Sarcoptes scabiei, a 0.3- to 0.5-mm mite that can burrow and deposit eggs in the human stratum corneum. Microscopic visualisation of mites, their eggs, or faeces in skin scrapings is helpful but not essential to initiation of treatment. Most popular treatment options include topical permethrin and oral ivermectin. Primarily considered a nuisance in the developed world. Children in the developing world can contract secondary streptococcal infection in their skin lesions, with potential complications of rheumatic heart disease or post-streptococcal glomerulonephritis.
150
Epidemiology scabies
The prevalence of scabies is estimated to be 300 million cases worldwide. However, this varies geographically and in some communities, particularly those that are sub-tropical and developing, prevalence may approach 50%.
151
RFs for scabies
``` STRONG Crowded living Contact with infected <15 or >65 Multiple partners Immunosuppression ``` ``` WEAK Poor hygiene Contact with animals Contaminated contact eg towels Winter season ```
152
Sx of scabies
``` COMMON Hx of itching across family Intense pruritus worse at night Burrows Papules, vesicles excoriations +ve ink burrow test Papules on face neck palms and soles Bullae Crusted lesions Breakfast-Lunch-dinner bite pattern ``` Violaceous pruritic nodules of the axillae, groin, and male genitalia. Represent a hypersensitivity reaction to mite antigens and may persist weeks or months after treatment.
153
Ix for scabies
Clinical or ectoparasite preparation
154
Rx of scabies
1. Permethrin or ivermectin permethrin topical: (5%) apply from neck down to the soles of feet, wash off after 8-14 hours; may need to repeat treatment after 10-14 days if evidence of live mites Secondary options ivermectin: 200 micrograms/kg orally as a single dose, followed by a second course of treatment in 10 days OR ivermectin topical: (0.5%) consult specialist for guidance on dose 2. If allergic to above -> malathion topical: (0.5%) apply to the whole body, wash off after 8-12 hours; reapply in 10-14 days Secondary options benzyl benzoate: consult product literature for guidance on dosage IF NORWEGIAN -> add keratolytic eg Urea topical. Used to enhance drug penetration in crusted scabies, where skin is typically hyperkeratotic.
155
Complications / prognosis scabies
post-scabetic pruritus Seconday bacterial infection Nodular scabies or pseudolymphoma
156
A 75-year-old nursing home resident presents with a 2-month history of intensely pruritic, erythematous papules on his torso, groin, and extremities. He states that the pruritus is worse at night. He was treated with topical steroids and oral antihistamines without improvement. Other nursing home residents have similar complaints.
scabies Other presentations Norwegian, or crusted, scabies is primarily seen in immunocompromised or institutionalised patients, or those with neurological disorders such as Down's syndrome. It is characterised by hyperkeratotic lesions, predominantly on the palms and soles, which contain thousands to millions of mites.
157
A 5-year-old boy who attends kindergarten presents with a 2-week history of generalised pruritus and erythematous papules in the finger web spaces, axillae, and groin. Due to his repetitive scratching, many of the lesions have secondary honey-coloured crusting. No other family members are affected.
scabies Other presentations Norwegian, or crusted, scabies is primarily seen in immunocompromised or institutionalised patients, or those with neurological disorders such as Down's syndrome. It is characterised by hyperkeratotic lesions, predominantly on the palms and soles, which contain thousands to millions of mites.
158
Define headlice / pediculosis capitis
The head louse (Pediculus humanus capitis) is an obligate ectoparasite that lives on human beings and feeds on human blood. Head lice infestation (pediculosis capitis) mainly affects those who are socially active, particularly young children. Head lice infestations are associated with little morbidity but cause strain and distress to parents, carers, and teachers. Effective pediculicides exist, but head lice in some areas have developed resistance to products with a neurotoxic mode of action. Products with a physical mode of action show promise as an alternative to neurotoxic treatments; they are nontoxic, and lice are less likely to develop resistance. Physicians should take an active role in the treatment of head lice infestations by being available to confirm active cases and being knowledgeable about first- and second-line treatment options in their communities. Knowledge of the life cycle of lice and the mode of transmission emphasise that 'no nit' policies in schools are not effective and should be abandoned.
159
Epidemiology of headlice / pediculosis capitis
Some examples include Australia (prevalence in school children of 13%, with a range between schools of 0% to 28%),[5] the UK (prevalence of 2%),[6] and China (prevalence of 14%, with a range of 0% to 52%).
160
RFs for headlice / pediculosis capitis
``` STRONG 3-12 Female Not black Close contact with infected Crowded living ``` WEAK Low socio Poor hygeine Contact with contaminated items
161
Sx of headlice / pediculosis capitis
COMMON Scalp pruritus Live nymphal or adult louse Eggs visible on hair shaft within 1cm of scalp Small red papule under hairline at nape of neck UNCOMMON Lymphadenopathy Erythema with honey coloured crust on scalp - Impetigo can accompany head lice infestations
162
Ix for headlice / pediculosis capitis
clinical
163
Rx of headlice / pediculosis capitis
dimeticone topical: apply to the hair and scalp as directed, leave for 30 minutes to 8 hours (depending on product) before rinsing; repeat treatment after 8-10 days OR permethrin topical: (1%) apply to the hair and scalp as directed, leave for 10 minutes before rinsing OR pyrethrin topical: apply to the hair and scalp as directed, leave for 10 minutes before rinsing Secondary options: benzyl alcohol lotion: infants ≥6 months of age: (5%) apply to the hair and scalp as directed, leave for 10 minutes before rinsing OR ivermectin topical: infants ≥6 months of age: (0.5%) apply to the hair and scalp as directed, leave for 10 minutes before rinsing OR spinosad topical: infants ≥6 months of age: (0.9%) apply to the hair and scalp as directed, leave for 10 minutes before rinsing Patients who cannot or do not want to use pediculicides with a neurotoxic mode of action, or run the risk of inducing resistance, can use mechanical removal.
164
Complications of headlice / pediculosis capitis
Imbetigo
165
Prognosis of headlice / pediculosis capitis
Recurrence of head lice infestation is not uncommon, and can occur for several reasons. Improper use of the original pediculicide or other method resulting in self-re-infestation from hatching eggs that were not killed or picked/combed out would be the most common cause. A second common cause would be re-infestation from a family member or other close contact that was not treated simultaneously with the index case. Re-infestation can also occur any time the index case comes into close contact with any other person with head lice infestation. None of these would necessarily indicate resistance to the original product.
166
A 7-year-old white girl presents to the school nurse. She was referred by her teacher, who noted she was scratching her head a lot. The school nurse knows there have been no active cases of head lice identified in this student's class recently, but the girl says she had been to a 'sleepover' about 1 month ago. The girl has shoulder-length hair, and claims she has never had head lice before. When the school nurse examines the nape of the girl's neck, she finds small whitish spots within 1 cm of the scalp that are firmly adhered to the hair shaft. As the nurse continues to examine the girl's scalp under bright light, she sees a sesame seed-sized insect-like object crawl quickly away from the area.
headlice / pediculosis capitis Other presentations An atypical presentation would involve a bacterial infection (i.e., scalp impetigo caused by streptococci and/or staphylococci) with posterior cervical lymphadenopathy, secondary to a longstanding infestation with excoriated skin from frequent scratching. This would be seen most commonly in situations where there is overcrowding (e.g., homeless shelters) or where access to proper treatment may be limited.
167
An 8-year-old white girl presents to be checked for lice because her mother is concerned she may have another infestation. She has treated her daughter with over-the-counter pediculicide 3 times in the past 2 months, but admits she is not sure what to look for - she treats when she hears about a case in her daughter's class. When examining her scalp under a bright light, dozens of small white objects firmly adherent to the hair shaft can be seen, but all are more than 1 cm from the scalp. Microscopic examination of one shows it to be an empty louse egg case, indicating that the girl most probably had an infestation 2 months ago that resolved.
headlice / pediculosis capitis Other presentations An atypical presentation would involve a bacterial infection (i.e., scalp impetigo caused by streptococci and/or staphylococci) with posterior cervical lymphadenopathy, secondary to a longstanding infestation with excoriated skin from frequent scratching. This would be seen most commonly in situations where there is overcrowding (e.g., homeless shelters) or where access to proper treatment may be limited.
168
Define erythroderma
Erythroderma is the term used to describe intense and usually widespread reddening of the skin due to inflammatory skin disease. It often precedes or is associated with exfoliation (skin peeling off in scales or layers), when it may also be known as exfoliative dermatitis (ED). Idiopathic erythroderma is sometimes called the ‘red man syndrome’.
169
Epidemiology of erythroderma
Erythroderma is rare. It can arise at any age and in people of all races. It is about 3 times more common in males than in females. Most have a pre-existing skin disease or a systemic condition known to be associated with erythroderma. About 30% of cases of erythroderma are idiopathic. Erythrodermic atopic dermatitis most often affects children and young adults, but other forms of erythroderma are more common in middle-aged and elderly people.
170
Aetiology of erythroderma
It is not known why some skin diseases in some people progress to erythroderma. The pathogenesis is complicated, involving keratinocytes and lymphocytes, and their interaction with adhesion molecules and cytokines. The result is a dramatic increase in turnover of epidermal cells. The most common skin conditions to cause erythroderma are: Drug eruption — with numerous diverse drugs implicated (list of drugs) Dermatitis especially atopic dermatitis Psoriasis, especially after the withdrawal of systemic steroids or other treatment Pityriasis rubra pilaris Other skin diseases that less frequently cause erythroderma may include: Other forms of dermatitis: contact dermatitis (allergic or irritant), stasis dermatitis (venous eczema) and in babies, seborrhoeic dermatitis or staphylococcal scalded skin syndrome Blistering diseases including pemphigus and bullous pemphigoid Sezary syndrome (the erythrodermic form of cutaneous T-cell lymphoma) Several very rare congenital ichthyotic conditions. Erythroderma may also be a symptom or sign of systemic disease. These may include: Haematological malignancies, such as lymphoma and leukaemia Internal malignancies, such as carcinoma of rectum, lung, fallopian tubes, colon, prostate (paraneoplastic erythroderma) Graft-versus-host disease HIV infection.
171
RFs for erythroderma
The most common skin conditions to cause erythroderma are: Drug eruption — with numerous diverse drugs implicated (list of drugs) Dermatitis especially atopic dermatitis Psoriasis, especially after the withdrawal of systemic steroids or other treatment Pityriasis rubra pilaris Other skin diseases that less frequently cause erythroderma may include: Other forms of dermatitis: contact dermatitis (allergic or irritant), stasis dermatitis (venous eczema) and in babies, seborrhoeic dermatitis or staphylococcal scalded skin syndrome Blistering diseases including pemphigus and bullous pemphigoid Sezary syndrome (the erythrodermic form of cutaneous T-cell lymphoma) Several very rare congenital ichthyotic conditions. Erythroderma may also be a symptom or sign of systemic disease. These may include: Haematological malignancies, such as lymphoma and leukaemia Internal malignancies, such as carcinoma of rectum, lung, fallopian tubes, colon, prostate (paraneoplastic erythroderma) Graft-versus-host disease HIV infection.
172
Sx of erythroderma
Erythroderma is often preceded by a morbilliform (measles-like) eruption, dermatitis, or plaque psoriasis. Generalised erythema can develop quite rapidly in acute erythroderma, or more gradually over weeks to months in chronic erythroderma. Signs and symptoms of erythroderma: By definition, generalised erythema and oedema or papulation affect 90% or more of the skin surface. The skin feels warm to the touch. Itch is usually troublesome and is sometimes intolerable. Rubbing and scratching leads to lichenification. Eyelid swelling may result in ectropion. Scaling begins 2-6 days after the onset of erythema, as fine flakes or large sheets. Thick scaling may develop on the scalp with varying degrees of hair loss including complete baldness. Palms and soles may develop yellowish, diffuse keratoderma. Nails become dull, ridged, and thickened or develop onycholysis and may shed (onychomadesis). Lymph nodes become swollen (generalised dermatopathic lymphadenopathy). Clues may be present as to the underlying cause. Serous ooze, resulting in clothes and dressings sticking to the skin and an unpleasant smell, is characteristic of atopic erythroderma. Persistence of circumscribed scaly plaques in certain sites such as elbows and knees suggests psoriasis. Islands of sparing, follicular prominence, orange-hue to keratoderma are typical of pityriasis rubra pilaris. Subungual hyperkeratosis, crusting on palms and soles, and burrows are indicative of crusted scabies. Sparing of abdominal creases (deck chair sign) is typical of papuloerythroderma of Ofuji. Systemic symptoms may be due to the erythroderma or to its cause. Lymphadenopathy, hepatosplenomegaly, abnormal liver dysfunction and fever may suggest a drug hypersensitivity syndrome or malignancy. Leg oedema may be due to inflamed skin, high output cardiac failure and/or hypoalbuminaemia.
173
Ix for erythroderma
The blood count may show anaemia, white cell count abnormalities, and eosinophilia. Marked eosinophilia should raise suspicions for lymphoma. > 20% circulating Sézary cells suggests Sézary syndrome C-reactive protein may or may not be elevated. Proteins may reveal hypoalbuminaemia and abnormal liver function. Polyclonal gamma globulins are common, and raised immunoglobulin E (IgE) is typical of idiopathic erythroderma. Skin biopsies from several sites may be taken if the cause is unknown. They tend to show nonspecific inflammation on histopathology. Diagnostic features may also be present. Direct immunofluorescence is of benefit if an autoimmune blistering disease or connective tissue disease is considered.
174
Rx of erythroderma
The following general measures apply: Discontinue all unnecessary medications Monitor fluid balance and body temperature Maintain skin moisture with wet wraps, other types of wet dressings, emollients and mild topical steroids Prescribe antibiotics for bacterial infection Antihistamines may or may not be helpful for the itch. If a cause can be identified then specific treatment should be started, such as topical and systemic steroids for atopic dermatitis; acitretin or methotrexate for psoriasis.
175
Prognosis / complications of erythroderma
Heat loss leads to hypothermia. Fluid loss leads to electrolyte abnormalities and dehydration. Red skin leads to high-output heart failure. A secondary skin infection may occur (impetigo, cellulitis). General unwellness can lead to pneumonia. Hypoalbuminaemia from protein loss and increased metabolic rate causes oedema. Longstanding erythroderma may result in pigmentary changes (brown and/or white skin patches).
176
Define impetigo
Impetigo is a superficial, contagious, blistering infection of the skin caused by the bacteria Staphylococcus aureus and Streptococcus pyogenes. It has two forms: non-bullous and bullous. Bullae are fluid-filled lesions of >0.5 cm in diameter. Non-bullous impetigo is the more common form (70% of cases).[1] Though bullae are not present in this form, vesicles (fluid-filled lesions <0.5 cm in diameter) may appear transiently early in the disease. Aetiological agents are Staphylococcus aureus, Streptococcus pyogenes, or a combination of the two organisms. Lesions occur at sites of skin trauma. Bullous impetigo is due to Staphylococcus aureus skin infection. Lesions occur on intact skin. Highly contagious and common bacterial infection of the skin that typically occurs in children; a key consideration for schools and playgroups. Typically staphylococcal or streptococcal. Diagnosis is usually clinical; bacterial skin cultures are reserved for extensive disease or where there is risk of spread of infection. Skin that has been broken by minor trauma or other disease is particularly susceptible to infection. Tends to resolve spontaneously or with topical antiseptics. Topical antibiotics (such as mupirocin or fusidic acid) are effective, but resistance may be an issue in some regions. Systemic antibiotics may be necessary; for example, if infection is recalcitrant to topical treatment, if there is concern about complications (e.g., acute rheumatic fever), or if deeper infection is suspected.
177
Epidemiology of impetigo
Impetigo is the most common bacterial skin infection in children worldwide, with a global prevalence of over 162 million The annual incidence of impetigo in the UK is approximately 80/100,000 in children aged 0 to 4 years, decreasing to approximately 50/100,000 in those aged 5 to 14 years. It decreases further in older age groups
178
RFs for impetigo
``` STRONG Increased humidity Poor hygeine Overcrowding Malnutrition Chronic Staph A colonisation Concomitant skin disease ```
179
Sx of impetigo
``` Vesicles/bullae Crusting Erythema Pruritus Pain Mucopurulent exudate Lymphadenopathy Fever ```
180
Ix for impetigo
Clinical Possible to do a skin culture - Investigate bacteriology when disease is extensive or if there is risk of spread of infection. In treatment-resistant disease, use to test for the presence of MRSA.
181
Rx of impetigo
NOT WIDESPREAD 1. TOPICAL ABx mupirocin topical: (2%) apply to affected skin and nares three times daily OR fusidic acid: (2%) apply to the affected skin three to four times daily OR retapamulin topical: (1%) apply to affected skin twice daily for 5 days OR ozenoxacin topical: (1%) apply to affected skin twice daily for 5 days Intranasal antibiotic (applied to the nares at the same frequency as antibiotic skin application for 5 to 7 days of the month) is additionally used for patients who experience frequent recurrences of their impetigo. WIDESPREAD: 1. ORAL ABx clindamycin: children: 10-30 mg/kg/day orally given in divided doses every 6-8 hours; adults: 150-450 mg orally every 6-8 hours OR trimethoprim/sulfamethoxazole: children: 8-10 mg/kg/day orally given in divided doses every 12 hours; adults: 160 mg orally every 12 hours More Secondary options doxycycline: children: 2.2 to 4.4 mg/kg/day orally given in divided doses every 12 hours; adults: 50-100 mg orally twice daily CONSERVATIVE In addition to topical antibiotic treatment, twice-daily washing with soap and water is good general skin care for all patients with impetigo, as it helps to reduce the amount of bacteria on the skin. For older children and adults, particularly those with more widespread skin involvement, chlorhexidine can be a good antibacterial cleanser to add to the routine. Chlorhexidine is not recommended in young children (due to issues related to its potential toxicity with absorption). This washing routine may also be recommended for close patient contacts who may have acquired the bacteria.
182
Prognosis / complications of impetigo
Patients typically need no follow-up unless they are: Neonates Impetigo can be more serious for neonates, with systemic and potentially life-threatening complications Need careful daily evaluation until clear of infectious lesions. Immunocompromised May be at risk of poor response to treatment, with extension of infection into deeper tissues Might eventually require treatment with parenteral antibiotic agents. ``` COMPLICATIONS Acute glomerulonephritis Cellulitis Osteomyelitis Sepsis ```
183
A 5-year-old boy presents with superficial erosions and crust in the peri-oral region for 3 days. He is generally well with no recent history of fever, sore throat, nausea, diarrhoea, or changes in appetite or energy level. There is a history of similar eruption in other children at his nursery.
impetigo
184
A 5-day-old newborn presents with a rapidly enlarging bulla with thin roof and a thin band of erythema around the bulla. The bulla is flaccid and contains clear fluid. Careful examination reveals an adjacent smaller bulla. The baby is well.
impetigo
185
Define lichen planus
Lichen planus (LP) is a pruritic, chronic inflammatory dermatosis resulting from keratinocyte apoptosis that affects the skin, mucous membranes, genitalia, scalp (lichen planopilaris), and nails. An idiopathic inflammatory disease affecting the skin, hair, nails, and mucous membranes, usually self-limiting in nature. Characteristic eruption consists of itchy, shiny, flat-topped violaceous papules and plaques favouring the extremities. White net-like patches or ulceration of mucous membranes, nail deformity, scarring alopecia, and other types of skin lesions may also occur. Most patients develop the disease between the age of 30 and 60 years. Hepatitis C infection may increase susceptibility, especially to oral disease. Treatment may involve corticosteroids, retinoids, calcineurin inhibitors, immunosuppressants, and phototherapy.
186
Epidemiology of lichen planus
Lichen planus (LP) usually develops in middle age between 30 and 60 years and shows a slight female preponderance.[2] In rare circumstances it may arise in childhood.[20][21] In the US, prevalence is estimated to be less than 1%.[2][22] In India, one study found that LP represents 0.38% of outpatient dermatology diagnoses.[23] Cutaneous LP is the most common presentation, with hyper-trophic and actinic cutaneous forms being most common in the Indian population.
187
Aetiology of lichen planus
There are no proven causes, but it is thought that autoimmunity is involved. Hepatitis C may also induce susceptibility by changing cytokine expression in some patient populations. Oral allergens may play a role in oral disease, and exogenous antigens, such as those found in tattoo ink, may encourage development at tattoo sites.
188
RFs for lichen planus
STRONG Hep C ``` WEAK Polymorphism in TNFa Hep B Influenza vaccination Psychosocial stress Oral allergens ```
189
Sx of lichen planus
``` COMMON Pruritus Violacious, topped papules or plaques Wickhams striae - wickham striae are whitish lines visible in the papules of lichen planus Mucosal erosions and lacy white network ``` ``` UNCOMMON Scarring alopecia Nail involvement Dorsal pterygium - Cuticle growth over the nail may be evident. History of hep C ```
190
Ix for lichen planus
Clinical diagnosis If very unclear - histopathology
191
Rx of lichen planus
1. Topical corticosteroid clobetasol topical: (0.05%) apply to the affected area(s) twice daily, maximum 50 g/week + Antihistamine diphenhydramine: 25-50 mg orally every 6-8 hours when required, maximum 300 mg/day 2. Oral corticosteroid or retinoid prednisolone: consult specialist for guidance on dose +/- phototherapy 3. Immunosuppressant - ciclosporin: 2.5 to 5 mg/kg/day orally given in 2 divided doses mycophenolate mofetil: 0.5 to 1 g orally twice daily
192
Prognosis of lichen planus
This chronic condition may remain unresponsive to treatment or may recur once treatments are discontinued. For these patients, symptom relief is important. Many of the treatment options have undesirable side effects that require consideration by practitioners and patients.
193
Complications of lichen planus
Scarring alopecia | SCC
194
A 50-year-old woman presents with persistent, intensely pruritic, cutaneous lesions on the flexor wrists and ankles. Eruption was spontaneous and treatment with over-the-counter hydrocortisone cream and emollients were unsuccessful. On examination there are several discrete, violaceous, polygonal, flat-topped papules and plaques with overlying white reticulated scale. Onychodystrophy is also apparent. No excoriation marks are visible. Examination of the mouth reveals lacy change in the left buccal mucosa.
lichen planus Lichen planus (LP) may present after hepatitis B and inactivated influenza vaccination. LP may also present as lichen planus pemphigoides. This is a relatively rare condition, distinct from bullous lichen planus, and represents an overlap between lichen planus and a group of sub-epidermal blistering diseases. A crossover condition of LP and lupus erythematosus has also been described. This may present as LP lesions in a photodistribution (at sites routinely exposed to the sun) with or without other signs of lupus erythematosus.
195
Define cutaneous vascular lesions
Cutaneous vascular lesions are the most common pediatric birthmarks. Flat vascular malformations tend to persist, but raised vascular lesions, known as hemangiomas, generally involute. Although not always necessary, treatment of flat lesions, if desired, is best accomplished with flash-lamp pumped pulsed dye laser. Therapy of hemangiomas varies depending on the presence of associated symptoms or syndromes. Specifically, hemangiomas that are likely to lead to loss of function or life (e.g., lesions of internal organs, lesions associated with coagulopathy) should be treated promptly. Treatment may also be required for hemangiomas that are likely to lead to scarring when the lesion involutes, such as hemangiomas of the nose and lip. The natural history of hemangiomas includes proliferative, stationary and involutional phases. Many superficial hemangiomas resolve with minimal sequelae
196
Which hemangiomas require Rx?
``` Threat to life or function Kasabach-Merritt syndrome (coagulopathy) Anatomic site Vision impairment Respiratory impairment High-output cardiac failure (mortality up to 50 percent)11 Hepatic lesions Other internal lesions Location in scarprone area Nose Lip Ear Glabellar area Any large facial hemangiomas Pedunculated lesions Tendency to bleed or to become infected Rapid rate of growth (tripling in size within weeks) ```
197
Define SJS and toxic epidermal necrolysis
Stevens-Johnson syndrome (SJS) is a severe skin detachment with mucocutaneous complications. It is an immune reaction to foreign antigens. SJS is a more severe form of erythema multiforme major and a less severe manifestation of toxic epidermal necrolysis (TEN). Classification is dependent on the percentage of skin involvement: SJS has <10% total body surface area (TBSA) involvement; SJS/TEN overlap has 10% to 30% TBSA involvement; and TEN has >30% TBSA involvement. Stevens-Johnson syndrome (SJS) and toxic epidermal necrolysis (TEN) may be associated with a preceding history of medication use, most commonly anticonvulsants, antibiotics, and non-steroidal anti-inflammatory drugs. Other associated factors include infections. The patient may present with Nikolsky's sign, where the epidermal layer easily sloughs off when pressure is applied to the blistered or erythematous area. Diagnosis is made by clinical presentation and confirmed with skin biopsy. On diagnosis the offending medicine should be stopped. Management is then supportive. Patients do best if they are sent to a burn centre for wound care as soon as the diagnosis is suspected or made. The majority of SJS patients recover (mortality 1% to 5%). SJS can recur either with the same medicine or with another medicine. TEN has a higher mortality (25% to 30%). In the long term, patients should ensure they are not re-exposed to the trigger medicine and be careful of self-medicating. They should avoid sunlight during healing and moisturise their skin. Currently, patients at risk may need human leukocyte antigen screening if they are to receive medications such as carbamazepine, allopurinol, or trimethoprim/sulfamethoxazole.
198
Epidemiology of SJS and toxic epidermal necrolysis
The incidence of SJS is estimated at 1 to 6 cases/million person-years, and 0.4 to 1.2 cases/million person-years for TEN, in all races and ages and both sexes.
199
Aetiology fo SJS and toxic epidermal necrolysis
Stevens-Johnson syndrome (SJS) may be a consequence of a disease process, or of an antibiotic, anticonvulsant, or other medicine. A thorough history is essential. Several aetiological factors exist. 1. Infection SJS can be a sequela of a number of conditions, including: ``` Upper respiratory tract infections Pharyngitis Otitis media Mycoplasma pneumoniae Herpes Epstein-Barr virus Cytomegaloviruses. 2. Vaccination ``` Smallpox vaccination can precipitate erythema multiforme or SJS. 3. Medicine The most frequent drugs implicated in SJS and toxic epidermal necrolysis (TEN) are: Anticonvulsants (e.g., carbamazepine, phenobarbital, phenytoin, valproic acid, lamotrigine) Antibiotics (e.g., sulfonamides, aminopenicillins, quinolones, cephalosporins) Antifungals Antiretrovirals (e.g., nevirapine, abacavir) and antivirals (e.g., telaprevir, aciclovir) Anthelmintics Analgesics (e.g., paracetamol) Non-steroidal anti-inflammatory drugs and selective COX-2 inhibitors Antimalarials Azathioprine Sulfasalazine Allopurinol Tranexamic acid Corticosteroids Psychotropic agents Chlormezanone Anticancer drugs (e.g., bendamustine, busulfan, chlorambucil) Retinoids.
200
RFs for SJS and toxic epidermal necrolysis
``` STRONG Anticonvulsants Recent infections Recent ABx Other medicines - Other medicines that may put the patient at risk are: antifungals, antivirals (e.g., telaprevir, aciclovir),[31] antiretrovirals (e.g., nevirapine, abacavir),[12][27][28] anthelmintics, analgesics (e.g., paracetamol),[26] non-steroidal anti-inflammatory drugs, selective COX-2 inhibitors,[29][30] antimalarials, corticosteroids,[10] azathioprine, trimethoprim/sulfamethoxazole, sulfasalazine,[35] allopurinol, tranexamic acid,[34] psychotropic agents,[32] chlormezanone,[19][36] anticancer drugs (e.g., bendamustine, busulfan, chlorambucil),[36] and retinoids.[37] SLE AIDs Radiotherpay Smallpox vaccination ``` WEAK BM transplant
201
Sx of SJS and toxic epidermal necrolysis
A sudden rash or a rash that develops after a new medicine is started. Erosions or ulceration of the eyes, lips, mouth, pharynx, oesophagus, gastrointestinal tract, kidneys, liver, anus, genital area, or urethra. More common in SJS than TEN. Nikolsky's sign - Epidermal layer easily sloughs off when pressure is applied to the affected area. More common in TEN than SJS. UNCOMMON Blisters
202
Ix for SJS and toxic epidermal necrolysis
Skin biopsy - The definitive test for diagnosis. It is necessary to get the skin biopsy as soon as possible if there is a suspicion that the patient may have SJS or TEN - keratinocyte apoptosis with detachment of the epidermal layer of the skin from the dermal layer Blood cultures - -ve FBC - normal unless coexistent sepsis Arterial blood gases and oxygen saturation will help to determine the patient's clinical respiratory status. One of the complications of SJS/TEN is mucosal involvement of the upper and lower respiratory tract, with vesicle formation, ulceration, and actual mucosal sloughing that may lead to laryngeal stridor, possible retractions, and oedema of the nasopharynx.
203
Rx of SJS and toxic epidermal necrolysis
Immediate withdrawal of causative agent Anticoagulation PPI Dressing + topical emollients and antibacterials All patients manifesting signs and symptoms of SJS/TEN should have an ophthalmological consultation and a full examination upon admission, in a bid to preserve vision and reduce complications Nutritional support For >40% TBSA, consider oxandrolone to reduce the loss of muscle mass and to promote weight gain and healing. oxandrolone: children: consult specialist for guidance on dose; adults: 2.5 mg orally two to four times daily for 2-4 weeks, maximum 20 mg/day Analgesia normal immunoglobulin human: children and adults: consult specialist for guidance on dose ciclosporin: children and adults: consult specialist for guidance on dose There have been sporadic case reports of successful treatment of TEN with ciclosporin. Patients can develop limitations in mobility with decreased strength. Arrange for daily exercises within the patient's capability with a physiotherapist and, if needed, an occupational therapist
204
Prognosis of SJS and toxic epidermal necrolysis
SJS The majority of patients recover, although they may develop some ocular or organ complications residual to the disease; the mortality is 1% to 5%. SJS/TEN overlap The majority of patients recover, although they may develop some complications of ocular or organ origin. TEN The majority of patients recover, although they may develop some ocular or organ complications residual to the disease; the mortality is 25% to 35%
205
Complications of SJS and toxic epidermal necrolysis
``` Dehydration Abnormal skin pigmentation Acute compartment syndrome Infection Ocular complications Nail plate loss Acute liver or renal failure Hypothermia Vaginal synechiae Pulmonary complications Ureteral perforation ```
206
A 27-year-old man with AIDS presents with a rash that developed over the last 4 days, after being on prophylactic doses of trimethoprim/sulfamethoxazole for 19 days for a cluster of differentiation 4 count of <200 lymphocytes. He has a 20% total body surface area (TBSA) skin slough with scattered patches throughout his torso and extremities. Except for conjunctival erythema, he does not have any mucosal involvement. Following successful treatment he is discharged after a 12-day hospital stay. Approximately 2 months after discharge, he takes 'antibiotics for a cold and oral thrush'. He develops a fever, diffuse skin erythema, and pruritus. He presents at hospital 4 hours after taking the medicine, with a blood pressure of 80/40 mmHg, temperature of 40°C (104°F), respiratory rate of 22 breaths per minute, and a heart rate of 96 bpm. It is discovered that the 'antibiotic' was trimethoprim/sulfamethoxazole.
SJS and toxic epidermal necrolysis Other highly specific features would be atypical target lesions (which may be confluent), macules, palpable purpuric lesions, or positive Nikolsky's sign (epidermal layer easily sloughs off when pressure is applied to the affected area). Among other presentations, the patient may have fever, malaise, ocular lesions, stomatitis, swelling of the tongue, diarrhoea, vomiting, dysuria, enlarged lymph nodes, arthralgias, arthritis, bronchitis, shortness of breath, wheezing, genital lesions, hypotension, and dehydration.
207
A 14-year-old boy presents with a 1-day history of fever, photophobia, chills, cough, tonsillar enlargement, and chapped, bleeding lips. He has a diffuse erythematous rash including the palms and soles of his feet, in total covering 57% TBSA. Within 24 hours the rash progresses to 87% TBSA involvement and the patient requires mechanical ventilation for respiratory distress. He has a history of asthma and has been taking ibuprofen for an upper respiratory infection that developed 2 weeks previously.
SJS and toxic epidermal necrolysis Other highly specific features would be atypical target lesions (which may be confluent), macules, palpable purpuric lesions, or positive Nikolsky's sign (epidermal layer easily sloughs off when pressure is applied to the affected area). Among other presentations, the patient may have fever, malaise, ocular lesions, stomatitis, swelling of the tongue, diarrhoea, vomiting, dysuria, enlarged lymph nodes, arthralgias, arthritis, bronchitis, shortness of breath, wheezing, genital lesions, hypotension, and dehydration.
208
A 14-year-old boy presents with a 1-day history of fever, photophobia, chills, cough, tonsillar enlargement, and chapped, bleeding lips. He has a diffuse erythematous rash including the palms and soles of his feet, in total covering 57% TBSA. Within 24 hours the rash progresses to 87% TBSA involvement and the patient requires mechanical ventilation for respiratory distress. He has a history of asthma and has been taking ibuprofen for an upper respiratory infection that developed 2 weeks previously.
SJS and toxic epidermal necrolysis Other highly specific features would be atypical target lesions (which may be confluent), macules, palpable purpuric lesions, or positive Nikolsky's sign (epidermal layer easily sloughs off when pressure is applied to the affected area). Among other presentations, the patient may have fever, malaise, ocular lesions, stomatitis, swelling of the tongue, diarrhoea, vomiting, dysuria, enlarged lymph nodes, arthralgias, arthritis, bronchitis, shortness of breath, wheezing, genital lesions, hypotension, and dehydration.
209
Define dyshidrotic dermatitis
Dyshidrotic dermatitis is a relatively common form of chronic dermatitis affecting the hands and feet. It is characterised by recurrent crops of 1- to 2-mm vesicles on the palms, soles, and lateral aspects of the fingers. Crops persist for 2 to 3 weeks and return at variable intervals. Pruritus accompanies the eruptions and may even precede them. Pompholyx is a term often used synonymously with dyshidrotic dermatitis, but it is better used to describe more acute, severe eruptions of large bullae on the hands and feet
210
Rx of dyshidrotic dermatitis
Avoidance of triggers or exacerbating factors identified for a specific patient. Advise frequent and liberal use of emollients on the affected areas, especially after any contact with water. Use white soft paraffin-based formulations, or a cream formulation if the patient does not tolerate this. Advise patients against prolonged wet work and suggest wearing protective gloves if necessary. Recommend use of mild cleansers only and avoidance of skin products with fragrances or dyes. Topical corticosteroids / immunomodulators clobetasol topical: (0.05%) apply sparingly to affected area(s) once daily at bedtime for up to 2 weeks, then switch to lower-potency corticosteroid Oral corticosteroids Antihistamines for itchiness Therapy for hyperhidrosis Some patients respond to low nickel diets Last line = phototherapy
211
Define discoid eczema
Discoid eczema, also known as nummular or discoid dermatitis, is a long-term (chronic) skin condition that causes skin to become itchy, swollen and cracked in circular or oval patches.
212
Define contact dermatitis
Contact dermatitis is an allergic or irritant skin reaction caused by an external agent. Also an allergic phenomenon but not IgE mediated - T cell mediated. IE type 4 hypersensitivity rather than type 1. Irritant contact dermatitis is caused by direct toxicity without prior sensitisation, and allergic contact dermatitis is a delayed hypersensitivity reaction. Results in localised burning, stinging, itching, blistering, redness, and swelling at the area of contact with the allergen or irritant. Patch testing may aid identification of the causative agent. Skin biopsy may also be helpful, but may not be able to distinguish between other causes of dermatitis. Treatment involves removal of the causative agent, future avoidance of the causative agent, topical corticosteroids, and/or a short course of oral corticosteroids. Rarely, contact dermatitis can become generalised, leading to areas of dermatitis in locations distant from the site of contact with the causative agent. Other allergic conditions may be triggered by exposure to an allergen, such as allergic rhinitis, asthma, and anaphylaxis.
213
Define seborrhoeic dermatitis
Seborrhoeic dermatitis (SD) is a common inflammatory skin disorder that usually manifests as erythema and scaling of the scalp, nasolabial folds, Chronic inflammatory skin disorder characterised by erythematous and greasy scaly patches. Patches are red, inflamed, and pruritic with micaceous scale. Circumscribed patches are found on the scalp, glabella, nasolabial fold, posterior auricular skin, and anterior chest. Variable course that seldom completely subsides. An infant form (cradle cap) usually resolves within the first few months of life. Tends to flare with stress. The adult scalp form is commonly termed dandruff or pityriasis capitis. Topical agents (corticosteroids, antifungals, calcineurin inhibitors, shampoos, and tar preparations) are used for mild and limited disease. Oral antifungals are reserved for severe and recalcitrant forms.
214
Sx of seborrhoeic dermatitis
``` Scalp itching / scalinf Glabellar scaling Nasolabial scaling Post auricular scaling White flakes Erythema ```
215
Sx of seborrhoeic dermatitis
``` Scalp itching / scalinf Glabellar scaling Nasolabial scaling Post auricular scaling White flakes Erythema ```
216
Rx of eczema herpeticum
In some cases it can be difficult to determine if the eczema has been infected by bacteria or viruses and both need to be treated. MDT: If there is any concern about possible eye involvement, an urgent ophthalmic opinion should be sought. Conservative  Monitor hydration status and fluid intake  Cool wet presses  Petroleum jelly Medical  IV acyclovir  IV antibiotics e.g co-amoxiclav  Analgesics  Antipyretics  Antihistamines, should be pre- scribed.  Steroids and other immunosuppressants (e.g. tacrolimus) should be discontinued (this is in contrast to bacterial infections alone when steroid creams are usually continued alongside the antibiotic). Topical treatments can usually be restarted after 1 week when the patient has improved.
217
Define eczema
Eczema is an inflammatory skin condition characterised by dry, pruritic skin with a chronic relapsing course. It can affect all age groups, but it is most commonly diagnosed before 5 years of age and affects 10% to 20% of children. Patients often have a personal or family history of other atopic diseases such as asthma or allergic rhinitis. Food allergies may occur at increased rates in this population. Eczema can be described as acute or chronic. Acute eczema is used to describe a flare-up of symptoms. Chronic is used to describe the condition when the patient develops signs of chronic inflammation (e.g., lichenification). The period of time before the condition is termed chronic is not clearly defined. Commonly presents with dry, itchy skin. Typically there is erythema, scaling, vesicles, or lichenification in skin flexures. Diagnosis is primarily clinical. First-line treatment is with emollients and topical corticosteroids. Other options that may be used in patients unresponsive to first-line therapy include topical calcineurin inhibitors, phototherapy, or immunosuppressive agents.
218
Epidemiology of eczema
The prevalence of eczema is 10% to 20% in US children, and 1% to 3% in adults Eczema usually presents in childhood, with 45% of patients diagnosed by 6 months of age, and 70% to 85% by 5 years of age. Remission is noted by 15 years of age in 60% to 70% of cases, although relapse may occur later in life
219
Aetiology of eczema
Eczema has a multifactorial aetiology, with a combination of genetic susceptibility and environmental factors contributing to disease development. Defects in the skin's barrier function and immune dysregulation following allergen exposure are thought to be key components in the development of this disease. Common loss-of-function variants of the epidermal barrier protein filaggrin are a major predisposing factor for eczema. Breaks in the epidermal barrier allow increased exposure and sensitisation to antigens. Mutations in genes that are crucial to normal epidermal barrier function have been identified and are thought to predispose patients to the development of eczema. Links have also been identified between eczema and areas of the genome that are known to encode cytokines and receptors involved in the Th2-mediated immune response that predominates in eczema
220
RFs of eczema
``` STRONG Age <5 Fix Allergic rhinitis Asthma Antihelmintic Rx in utero Active + passive exposure to CS ```
221
Sx of eczema
COMMON Pruritus Xerosis (dry skin) Sites: Infants typically show involvement of the cheeks, forehead, scalp, and extensor surfaces. Affected skin is often oedematous, with prominent weeping and crusting. Children typically have involvement of flexures, particularly the wrists, ankles, and antecubital and popliteal fossae. In addition to the areas affected by acute disease, chronic eczema often affects the neck, upper back, and arms, as well as the hands and feet. ``` Erythema Scaling Vesicles Papules Keratosis pillars Excoriations Lichenification Hypopigmentation ```
222
Rx of eczema
Emollients Intermittent topical corticosteroids hydrocortisone topical: (0.2 to 2.5%) apply sparingly to the affected area(s) twice daily Options include:[58] Low-potency: hydrocortisone, desonide.[59] Mid-potency: fluticasone, triamcinolone, fluocinolone. High-potency: mometasone, betamethasone, desoximetasone. Very high-potency: clobetasol, ulobetasol, diflorasone +/- antihistamine +/- oral Abs if infected CHRONIC: Can use immunosuppressants eg tacrolimus topical or crisaborole UV therapy possible Last line = systemic immunosuppressant: ciclosporin: adults: children: consult specialist for guidance on dose; adults: 2.5 to 5 mg/kg/day orally given in 2 divided doses Or even dupilimab = anti IL4/13 R
223
Prognosis of eczema
Approximately 60% of children will have symptom resolution as they enter puberty, but relapse may occur in 50% Many patients with milder disease are able to be maintained on emollient treatment with intermittent use of other topical agents during flares. Patients with more severe disease often require combination treatment that includes coal tar, ultraviolet light therapy, and systemic immunosuppressants.
224
Complications of eczema
Psychological stress Systemic SEs of corticosteroids Malignancy with tacrolimus Systemic effects of cyclosporin - hypertension and renal dysfunction Bacterial infection Methrotrexate SEs - Side effects include nausea, elevated liver enzymes, and occasionally pancytopenia or hepatic or pulmonary toxicity. Azathioprine SEs - Side effects include hepatotoxicity and myelotoxicity Eczema herpeticum - This is a severe skin infection caused by herpes simplex virus in a patient with eczema. Characteristic lesions are grouped vesicles or pustules, and may later progress to 'punched out' ulcerations. Patients often have extensive skin involvement, although areas of active dermatitis seem to be most severely affected
225
A 12-year-old female presents with dry, itchy skin that involves the flexures in front of her elbows, behind her knees and in front of her ankles. Her cheeks also have patches of dry, scaly skin. She has symptoms of hay fever and has recently been diagnosed with egg and milk allergy. She has a brother with asthma and an uncle and several cousins who have been diagnosed with eczema.
eczema Although eczema presents before the age of 5 in 70% to 85% of cases, the disease can affect any age. Adult patients with eczema often present with more lichenified skin changes, often affecting flexural areas, as well as the neck, upper back, and arms.
226
Ix for eczema
Clinical Could allergy test or measure IgE Skin biopsy possible
227
Ix for eczema
Clinical Could allergy test or measure IgE Skin biopsy possible
228
Define epidermoid cyst
An epidermoid cyst or epidermal inclusion cyst[1] is a benign cyst usually found on the skin. The cyst develops out of ectodermal tissue. Histologically, it is made of a thin layer of squamous epithelium. In contrast to pilar cysts, epidermoid cysts are usually present on parts of the body with relatively little hair.
229
Rx of epidermoid cyst
In case of fronto-ethmoidal epidermoid cysts, surgical resection appears to be the mainstay of treatment; however, the extent of resection is dictated by adherence of the tumor capsule to the surrounding vital structures H2O2 can be used
230
Define pilar cyst
A trichilemmal cyst, is a common cyst that forms from a hair follicle. They are most often found on the scalp. The cysts are smooth, mobile and filled with keratin
231
Rx of pilar cyst
Surgical excision | or punch biopsy technique - pulled out through punch biopsy hole
232
Rx of pilar cyst
Surgical excision | or punch biopsy technique - pulled out through punch biopsy hole
233
Rx of erythema multiforme
``` Emollient Topical or oral corticosteroids Sterile dressing Analgesia IF HSV -> aciclovir IF mycoplasma pneumonia -> doxy or macrolide ```
234
Prognosis of erythema multiforme
EM is usually a mild self-limiting disease with the lesions healing within 2 to 3 weeks without scarring. Recurrences are not generally frequent or life-threatening. However, suppressive therapy should be offered for prevention. Twice-daily dosing may maintain a better steady state of drug, but has poorer compliance in suppressive therapy.
235
Complications of erythema multiforme
Secondary bacterial infection
236
Rx of erythema nodosum
1. Conservative Bed rest Leg elevation Rx of underlying cause Analgesia 2. Postassium iodide potassium iodide: 300 mg (6 drops) orally three times daily 3. Intralesional corticosteroid injection: triamcinolone acetonide: consult specialist for guidance on intralesional dose 4. Systemic corticosteroids
237
Prognosis of erythema nodosum
Patients usually recover spontaneously within 1 to 2 months, depending on severity, and there is usually no scarring. Older patients with venous insufficiency may have lasting erythematous swelling of their ankles.
238
Complications of erythema nodosum
Ulceration /infection
239
An otherwise healthy, white, 10-year-old boy complains of a sore throat. He is running a low-grade fever of 38.0°C (100.6°F). Red lesions have developed on his extremities, and he is having some difficulty breathing after exertion. Oral erosions have limited his food and fluid intake. On physical examination, there are target lesions on his upper and lower extremities with erosions on his lower lip and palate. High-pitched wheezing is heard over the lower lungs bilaterally, and the patient is in mild discomfort when sitting upright. He is unable to open his mouth fully due to crusted oral lesions.
erythema multiforme Mucosal lesions of the mouth (60%-70%), nose, and genitalia may occur, and lesions can occasionally be associated with systemic lupus erythematosus (Rowell's syndrome). Although rare, ocular manifestations would be more likely to occur with EM major. Abdominal cramping and symptoms of upper respiratory infection can occur prodromally. Rare reports indicate that patients with HIV or cancer may present as EM.[6][7][8]
240
A 23-year-old man presents with a history of recurrent erythematous lesions over the extremities with each outbreak of herpes labialis. The episodes have become more frequent, and each recurrence is more severe with time. He was prescribed oral valaciclovir 3 weeks previously, to be taken at the first sign of an oral herpes simplex virus outbreak. On examination he has target lesions over the extremities and a mild cold sore over his upper lip.
erythema multiforme Mucosal lesions of the mouth (60%-70%), nose, and genitalia may occur, and lesions can occasionally be associated with systemic lupus erythematosus (Rowell's syndrome). Although rare, ocular manifestations would be more likely to occur with EM major. Abdominal cramping and symptoms of upper respiratory infection can occur prodromally. Rare reports indicate that patients with HIV or cancer may present as EM.[6][7][8]
241
A 25-year-old otherwise healthy woman awakens with a mild sore throat, fatigue, pain in both ankles, and red lesions on her legs. She reports weakness and fatigue for the past 2 days, but denies fever, chills, nausea, or night sweats. There is no history of using recreational drugs, oral contraceptives, or other medications. She has not travelled abroad for 3 years. Physical examination reveals red, tender, fixed, deep-seated nodules on both shins. Her ankles are mildly swollen and tender and she reports difficulty bearing weight. Chest, cardiovascular and ophthalmological examinations are normal. A chest x-ray shows left lower lobe infiltration. Her tuberculin skin test is negative. She is diagnosed with pneumococcal pneumonia.
Erythema nodosum
242
A 28-year-old woman awakens with red eyes, photophobia, and nodules on her legs. She denies fever, malaise, cough, or joint pains. She enjoys dairy products, but denies recent consumption of raw milk or cheese. She does not take recreational drugs. Physical examination shows bilateral iritis and enlarged lachrymal glands. Chest and cardiovascular examinations are normal, though the skin examination reveals many dark red nodules on her legs and lower thighs. These skin nodules are tender and non-mobile. Her tuberculin test is negative, but a chest x-ray shows bilateral hilar adenopathy consistent with the diagnosis of sarcoidosis.
Erythema nodosum
243
A 28-year-old woman awakens with red eyes, photophobia, and nodules on her legs. She denies fever, malaise, cough, or joint pains. She enjoys dairy products, but denies recent consumption of raw milk or cheese. She does not take recreational drugs. Physical examination shows bilateral iritis and enlarged lachrymal glands. Chest and cardiovascular examinations are normal, though the skin examination reveals many dark red nodules on her legs and lower thighs. These skin nodules are tender and non-mobile. Her tuberculin test is negative, but a chest x-ray shows bilateral hilar adenopathy consistent with the diagnosis of sarcoidosis.
Erythema nodosum
244
Rx of lipoma
Observation Surgical excision Liposuction - Alternative to surgical resection if the lipoma is soft and has a small connective tissue component Injection of corticosteroids or phosphatidylcholine may be used to trigger lipolysis. Symptomatic GI lipoma - open or LAP excision
245
Prognosis of lipoma
Superficial cutaneous lipomas Untreated, cutaneous lipomas tend to slowly increase in size or remain static. If excised, the majority heal without incident. There is a 1% to 2% recurrence rate, and these may require re-excision if the lesion increases in size or is symptomatic. Lesions treated with liposuction and lipolysis have a higher recurrence rates due to incomplete removal of the original lesion.
246
Complications of lipoma
``` Wound infection Serum Haematoma Nerve injury Vascular compromise Keloid ```
247
A 55-year-old woman presents with a right flank mass. She states she was recently diagnosed with diabetes mellitus, which she has been able to control with diet modifications. She lost 9 kg (20 pounds) within 3 months and then noticed a mass over her right lower rib cage. She denies pain but does report discomfort when she wears a jogging bra. On physical examination, the mass is soft, superficial, and mobile, and it measures 5 cm in diameter.
lipoma Other presentations Lipomas can present in locations other than subcutaneously on the trunk or proximal extremities. GI lipomas occur as submucosal lesions, most commonly in the oesophagus, stomach, and small intestine. This type may present with intestinal obstruction or bleeding. Rarely, lipomas can also occur in locations such as the adrenal glands, parotid glands, parapharyngeal space, breast, mediastinum, pleura, major airway, heart, superior vena cava, brain, and intraspinal areas. They can also occur on a hereditary basis in patients with familial multiple lipomatosis.[8][9] Patients with this autosomal condition tend to be male and have multiple, widespread, symmetric lipomas of the extremities and trunk.[2][10] Other hereditary syndromes that involve lipomas include Madelung disease, which is associated with men with heavy alcohol consumption and features benign symmetric lipomatosis of the head, neck, shoulders, and proximal upper extremities,[2][11] and Dercum's disease, also known as adiposis dolorosa, which occurs in middle-aged women and is characterised by painful lipomas on the trunk, shoulders, arms, and legs.[12] Angiolipomas present as painful, subcutaneous nodules, usually in young adults, and are multiple in more than 50% of cases.[2][3][13] They are composed of adipocytes interspersed with clusters of capillaries containing fibrin thrombi. Spindle cell lipomas, often seen in men between the ages of 45 and 65 years, occur in the posterior neck and shoulder area.[13] They are characterised by mature fat being replaced by collagen-forming spindle cells.[14][15] Intramuscular lipomas, which are usually poorly circumscribed and infiltrative, typically present in mid-adult life as slow-growing, deep masses located in the thigh or trunk. It is important to exclude an atypical lipomatous tumour or well-differentiated liposarcoma, as these are more common than an intramuscular lipoma in this anatomical position.[13][16] These latter diagnoses should also be considered in cases of retroperitoneal lesions. Hibernomas may arise in the trunk, retroperitoneum, and extremities and resemble the glandular brown fat found in hibernating animals.[5][13] They have a greater tendency to bleed during excision and recur if not fully excised.
248
A 35-year-old man presents with a right thigh nodule and a recurrent left chest wall nodule at the site of a prior scar. He states that he noticed a bump on his right lateral thigh 2 years previously and that the left chest wall lesion had been removed in clinic 3 years prior. The nodules have grown slightly over recent months. He also states that they bother him when he touches them. On physical examination, the nodules are 1 cm x 2 cm, soft, and mobile, and they feel subcutaneous.
lipoma Other presentations Lipomas can present in locations other than subcutaneously on the trunk or proximal extremities. GI lipomas occur as submucosal lesions, most commonly in the oesophagus, stomach, and small intestine. This type may present with intestinal obstruction or bleeding. Rarely, lipomas can also occur in locations such as the adrenal glands, parotid glands, parapharyngeal space, breast, mediastinum, pleura, major airway, heart, superior vena cava, brain, and intraspinal areas. They can also occur on a hereditary basis in patients with familial multiple lipomatosis.[8][9] Patients with this autosomal condition tend to be male and have multiple, widespread, symmetric lipomas of the extremities and trunk.[2][10] Other hereditary syndromes that involve lipomas include Madelung disease, which is associated with men with heavy alcohol consumption and features benign symmetric lipomatosis of the head, neck, shoulders, and proximal upper extremities,[2][11] and Dercum's disease, also known as adiposis dolorosa, which occurs in middle-aged women and is characterised by painful lipomas on the trunk, shoulders, arms, and legs.[12] Angiolipomas present as painful, subcutaneous nodules, usually in young adults, and are multiple in more than 50% of cases.[2][3][13] They are composed of adipocytes interspersed with clusters of capillaries containing fibrin thrombi. Spindle cell lipomas, often seen in men between the ages of 45 and 65 years, occur in the posterior neck and shoulder area.[13] They are characterised by mature fat being replaced by collagen-forming spindle cells.[14][15] Intramuscular lipomas, which are usually poorly circumscribed and infiltrative, typically present in mid-adult life as slow-growing, deep masses located in the thigh or trunk. It is important to exclude an atypical lipomatous tumour or well-differentiated liposarcoma, as these are more common than an intramuscular lipoma in this anatomical position.[13][16] These latter diagnoses should also be considered in cases of retroperitoneal lesions. Hibernomas may arise in the trunk, retroperitoneum, and extremities and resemble the glandular brown fat found in hibernating animals.[5][13] They have a greater tendency to bleed during excision and recur if not fully excised.
249
A 35-year-old man presents with a right thigh nodule and a recurrent left chest wall nodule at the site of a prior scar. He states that he noticed a bump on his right lateral thigh 2 years previously and that the left chest wall lesion had been removed in clinic 3 years prior. The nodules have grown slightly over recent months. He also states that they bother him when he touches them. On physical examination, the nodules are 1 cm x 2 cm, soft, and mobile, and they feel subcutaneous.
lipoma Other presentations Lipomas can present in locations other than subcutaneously on the trunk or proximal extremities. GI lipomas occur as submucosal lesions, most commonly in the oesophagus, stomach, and small intestine. This type may present with intestinal obstruction or bleeding. Rarely, lipomas can also occur in locations such as the adrenal glands, parotid glands, parapharyngeal space, breast, mediastinum, pleura, major airway, heart, superior vena cava, brain, and intraspinal areas. They can also occur on a hereditary basis in patients with familial multiple lipomatosis.[8][9] Patients with this autosomal condition tend to be male and have multiple, widespread, symmetric lipomas of the extremities and trunk.[2][10] Other hereditary syndromes that involve lipomas include Madelung disease, which is associated with men with heavy alcohol consumption and features benign symmetric lipomatosis of the head, neck, shoulders, and proximal upper extremities,[2][11] and Dercum's disease, also known as adiposis dolorosa, which occurs in middle-aged women and is characterised by painful lipomas on the trunk, shoulders, arms, and legs.[12] Angiolipomas present as painful, subcutaneous nodules, usually in young adults, and are multiple in more than 50% of cases.[2][3][13] They are composed of adipocytes interspersed with clusters of capillaries containing fibrin thrombi. Spindle cell lipomas, often seen in men between the ages of 45 and 65 years, occur in the posterior neck and shoulder area.[13] They are characterised by mature fat being replaced by collagen-forming spindle cells.[14][15] Intramuscular lipomas, which are usually poorly circumscribed and infiltrative, typically present in mid-adult life as slow-growing, deep masses located in the thigh or trunk. It is important to exclude an atypical lipomatous tumour or well-differentiated liposarcoma, as these are more common than an intramuscular lipoma in this anatomical position.[13][16] These latter diagnoses should also be considered in cases of retroperitoneal lesions. Hibernomas may arise in the trunk, retroperitoneum, and extremities and resemble the glandular brown fat found in hibernating animals.[5][13] They have a greater tendency to bleed during excision and recur if not fully excised.
250
Define melanoma
Melanoma is a malignant tumour arising from melanocytes. It is among the most common forms of cancer in young adults and typically presents as a new or changing deeply pigmented skin lesion. Lesions are often on sun-exposed anatomic locations in fair-skinned people. Diagnosis is by physical examination and confirmed by histopathological analysis of a biopsy. Early-stage melanoma has an overall survival rate of nearly 100%, while metastatic melanoma can be rapidly fatal. Incidence is increasing, with a lifetime risk in the US of 1 in 35 in men and 1 in 54 in women. Melanoma is the third most common skin cancer, but is the most common cause of skin cancer-related death. Up to 20% of patients develop metastatic disease. The prognosis depends on the stage at diagnosis: 99% of patients with in-situ melanoma will be cured with simple excision, while less than 10% of patients with metastatic disease will live beyond 5 years, although this statistic will change with the current era of new therapeutic agents. Recent advances in the understanding of tumour biology and immune regulation have led to the development of systemic targeted agents that have changed clinical practice, with further improvements expected with new compounds and combinations.
251
Epidemiology of melanoma
17 per 100k in the UK
252
Aetiology of melanoma
Melanoma arises from melanocytes, the pigment-producing cells found in the skin, eye, and CNS. Aetiology relates to both genetic and environmental factors. Genetic factors include the inheritance of sun-sensitive skin (fair skin type and susceptibility to sunburn) and specific melanoma-related genes. The major susceptibility gene associated with familial melanoma is CDKN2A, which encodes the P16 and p14ARF proteins. These proteins affect the p53 and retinoblastoma (Rb) cell cycle genes. DNA repair defects (as in xeroderma pigmentosum) are also implicated. Environmental factors include excessive exposure to solar and artificial UV radiation (e.g., tanning beds) and proximity to the equator. History of sunburns and intermittent high-intensity sun exposure are more strongly associated with melanoma development than cumulative chronic sun exposure.[ Phenotypic expressions of gene and environment interactions include the presence of naevi, ephelides (tanned macules) or freckles, and personal history of melanoma. It is estimated that only 25% to 42% of melanomas arise in pre-existing naevi.
253
RFs for melanoma
``` STRONG FHx PMHx Hx atypical naevi Fitzpatrick T1/2 Red or blonde hair High freckle density Sun exposure Sunbed Light eye colour >50 naevi Large naevi Immunosuppressant Xeroderma pigmentosum ```
254
Sx of melanoma
ABCDE evaluation - The mnemonic stands for: Asymmetry of the lesion, Border irregularity, Colour variability, Diameter >6 mm, Evolution. Ugly duckling Loss of pigmentation Spontaneous bleeding Constitutional Sx - Weight loss, fatigue, night sweats, headache, or cough may be symptoms of systemic metastasis in a patient with a history of melanoma. A bluish-white veiled appearance within a melanocytic lesion corresponds to dermal fibrosis, with pigmented melanophages in the dermis on histological examination of the pigmented lesion. These features represent regression and are common features of melanoma. Melanonychia striata Fixed lymphadenopathy in a patient with a history of melanoma, especially in lymph node basins draining the site of melanoma excision, are concerning for metastasis. Subcutaneous masses between the primary melanoma site and the draining lymph node basins should raise suspicion of in-transit metastases.
255
Ix for melanoma
Dermatoscopy - The criteria evaluated in dermatoscopy include the presence or absence and regularity of pigment networks, dots/globules, streaks, blue-white veils, blotches, comedo-openings, leaf-like pigmentation, red-blue lacunas, and pattern of vascular structures within pigmented lesions Skin biopsy - abnormal melanocytic proliferation in the epidermis and/or dermis typical of melanoma ``` May need sentinel node biopsy CXR CT MRI BRAF mutation analysis (MAPK Rx) ```
256
Rx of melanoma
Surgical excision + lymph node biopsy After excision: imiquimod topical: (5%) apply to the affected area(s) three to seven times weekly; consult specialist for further guidance on dose ``` STAGE 3: Ipilumab Clinical trial interferon alfa 2b or peginterferon Regional node radiation ``` STAGE 4 Targeted therapy for unresectable BRAF mutant melanoma Systemic chemo Clinical trial
257
Prognosis of melanoma
Approximate 5-year survival rates by stage: ``` Stage 0 (in-situ melanoma): >98% Stage I (Breslow's depth <1 mm and no nodal or metastatic disease): 90% to 95% Stage II (localised disease, intermediate to thick depth): 45% to 78% (78% for non-ulcerated melanoma of depth 1 to 4 mm; 45% for an ulcerated melanoma >4 mm depth) Stage III (nodal metastases): 69% (non-ulcerated melanoma of any depth with a single positive node) to 26% (ulcerated melanoma of any depth with 4 or more positive nodes) Stage IV (metastatic): without treatment, overall prognosis is bleak, ranging from 3% to 10% depending on the extent and sites of metastasis. New drugs are changing this, with 20% having long-term survival with ipilimumab, and newer drugs await long-term follow-up data. ```
258
Complications of melanoma
Local surgical sequelae Complications of sentinel node biopsy Local recurrence Metastatic disease
259
A 36-year-old fair-skinned woman presents with a dark, irregular, pigmented patch that she noted 4 months ago on the right posterior calf. It has gradually increased in size. She reports approximately 6 sunburns in the past and has been using tanning beds several times yearly for the past few years. Family history is positive for melanoma in her uncle. On physical examination, approximately 15 normal-appearing naevi, ranging in size from 3 mm to 5 mm, with symmetry, uniform brown coloration, and regular borders are noted elsewhere on the torso and extremities. The pigmented lesion on the right calf is asymmetrical along 2 axes, measures 1 cm x 0.8 cm, is deeply pigmented with several shades of brown, and has a jagged border. The popliteal nodes are not clinically palpable.
melanoma
260
A 51-year-old man presents with a large, dark, bleeding nodule on his back. He reports that it has been present and growing in size for at least 2 years, but he did not seek medical attention until now. On examination, a 2-cm ulcerated black nodule with an irregular border is present overlying the left posterior scapula. The left axillary nodes are clinically palpable.
melanoma
261
Rx of molluscum contangiosum
Observation (unless immunocompromised / stigma) 2. Curettage or cryotherapy 3. pulse dye laser therapy 4. cidofovir topical: (3%) product needs to be specially compounded as it is not available as a proprietary product
262
Prognosis of molluscum contangiosum
The prognosis of patients with molluscum contagiosum infections who are otherwise healthy is extremely good. Infection is expected to clear in 1 to 2 years and serious side effects are rare. Localised erythema and pruritus is a problem in one third of patients and may require therapy. In the absence of bacterial superinfection, molluscum lesions are generally tolerable. Immunocompromised patients may manifest hundreds of lesions and rapid spread, depending on their level of immunosuppression. In the case of HIV infection, spontaneous clearance is highly unlikely.
263
Complications of molluscum contangiosum
``` Atopic dermatitis Dyspigmentation Pitted scarring ID reaction - Id reactions associated with molluscum appear as lichenoid papules on the extensor surfaces, which are pruritic and eccentric to the site of molluscum. Erythema multiforme Bacterial superinfection ```
264
A 5-year-old white boy presents with a history of 2 months of bumps in the left axilla. Initially there was just one lesion; now the parents note that the child has half a dozen lesions. Some of the areas have been inflamed, and the child has pruritus, which keeps him up at night. One of his cousins, with whom he swam, may have such lesions as well. The child also suffers from seasonal allergies.
molluscum contangiosum Atypical or uncommon presentations include id reactions and erythema multiforme-like reactions, both of which are hypersensitivity responses. Abscesses form in about 1% as part of the immune response or due to true bacterial superinfection. Giant molluscum are nodular or tumour-like lesions that lack a central dell and are seen in areas of maceration, such as the intergluteal fold or sole of the foot; this presentation is quite rare. Unusual locations have been noted; the mucosal or conjunctiva lesions look similar to cutaneous lesions, but may have less prominence to the central dell. Occasionally, lesions can be seen in newborns or early infancy that are likely to be vertically transmitted from an infected female genital tract.[2]
265
A sexually active female university student presents complaining of itchy growths on the pubic area and the inner thighs, of 1 month's duration. The patient has been using a condom when she is sexually active with her boyfriend of 3 months. She is seen every 6 months by her gynaecologist and had a negative cervical smear 2 months ago. Pearly papules with a central dell can be observed on the pubic area and inner thighs. Some have surrounding erythema and excoriations.
molluscum contangiosum Atypical or uncommon presentations include id reactions and erythema multiforme-like reactions, both of which are hypersensitivity responses. Abscesses form in about 1% as part of the immune response or due to true bacterial superinfection. Giant molluscum are nodular or tumour-like lesions that lack a central dell and are seen in areas of maceration, such as the intergluteal fold or sole of the foot; this presentation is quite rare. Unusual locations have been noted; the mucosal or conjunctiva lesions look similar to cutaneous lesions, but may have less prominence to the central dell. Occasionally, lesions can be seen in newborns or early infancy that are likely to be vertically transmitted from an infected female genital tract.[2]
266
Define bullous pemphigoid
Bullous pemphigoid is a chronic, acquired autoimmune blistering disease characterised by auto-antibodies against hemidesmosomal antigens, resulting in the formation of a sub-epidermal blister. Typically occurs in older people and has a distinctive clinical appearance. There are several clinical variants, 1 of which occurs in childhood. In the prodromal, non-bullous phase, pruritus of variable intensity may be accompanied by eczematous or urticarial lesions for weeks or months. In the bullous stage, characteristic, tense vesicles or bullae develop on apparently normal or erythematous skin of the pre-existing eczematous or urticarial eruption. If the blisters burst, the eroded, crusty areas slowly heal to leave post-inflammatory hyperpigmentation. The lesions are usually symmetrical and favour the flexural aspects of the extremities, lower trunk, and abdomen. The treatment goal is to decrease or stop blister formation, to promote healing of existing blisters and erosions, and to control the associated pruritus. Topical and systemic corticosteroids, as well as other immunosuppressive and anti-inflammatory agents, are used.
267
Epidemiology of bullous pemphigoid
Average age onset 80-89 M>W Rare in children
268
Aetiology of bullous pemphigoid
Auto-antibodies are directed against 2 hemidesmosomal proteins, designated BP180 and BP230. Antibody binding to various proteins within this complex results in dermal-epidermal separation and tense blister formation.
269
RFs for bullous pemphigoid
STRONG 60-90yo DQB1*0301 WEAK Male
270
Sx of bullous pemphigoid
COMMON Pruritus TENSE blisters - pemphigoiD Deep - sub-epidermal - Don't break Blisters typically are found on the flexor surfaces of the arms and legs, axillae, groin, and abdomen. Patients may also present with urticarial plaques that may evolve into blisters. Oral lesions in 30%
271
Ix for bullous pemphigoid
Skin biopsy - sub-epidermal blister with dermal inflammatory cell infiltrate rich in eosinophils Immunoflourescent staining - linear band of IgG and/or C3 (and rarely other Ig classes) along the basement membrane zone
272
Rx of bullous pemphigoid
Localised lesions - topical corticosteroids Itch is usually controlled with the use of oral sedating antihistamines, such as hydroxyzine or diphenhydramine. Widespread: Oral corticosteroids + nicotinamide: 500-2000 mg orally once daily at bedtime + tetracycline: 250-500 mg orally four times daily Immunosuppression with cyclosporin
273
Prognosis of bullous pemphigoid
Most patients go into clinical remission with appropriate treatment. Nevertheless, mortality is considerable among older people, with an estimated death rate between 6% and 41% in the first year.
274
Complications of bullous pemphigoid
Osteoporosis from steroids Secondary infection Death
275
A 68-year-old man presents with a 3-month history of an itchy red rash on his torso and extremities. Over the past week, he has developed increasing numbers of tense, fluid-filled blisters on the inner arms and legs. Some of the blisters become bloody within a few days, then dry up and scab over, while others burst to produce raw, moist erosions. The itch is severe and wakes him up at night. He has no other complaints.
bullous pemphigoid Occasionally, blisters are limited to 1 body area and remain localised, such as on the shins (pretibial pemphigoid) or in the form of blisters on the palms and soles (dyshidrosiform pemphigoid). The disorder may present with small, tense, clustered vesicles only several millimetres in diameter (vesicular pemphigoid). Rarely, individual lesions are nodular, mimicking prurigo nodularis (nodular pemphigoid). If the initial, prodromal eruption involves the entire skin, the patient presents with erythroderma (erythrodermic pemphigoid) and subsequently develops isolated bullae. All these atypical presentations and variants are commonly accompanied by pronounced pruritus and tense blisters that last for days before rupturing. Oral lesions are present in up to one third of all patients, but seldom produce a functional deficit.[4] The generalised form is rare and may present much like psoriasis or atopic dermatitis, which may develop into vesicles or bullae. The urticarial form initially presents as itchy areas that then become bullae. However, sometimes bullae do not form. There is also a childhood form that is associated with vaccinations, in which bullae appear on the face, palms, and soles of the feet.
276
Define pemphigus
Pemphigus describes a group of autoimmune blistering diseases that involve the epidermal surfaces of the skin, mucosa, or both. There are 3 broad categories: pemphigus vulgaris (PV), pemphigus foliaceus (PF), and paraneoplastic pemphigus (PNP). Pemphigus is mediated by an autoantibody that binds a specific component of the desmosomal plaque. The desmosomal plaque holds the keratinocytes of the skin and mucosa together. Group of autoimmune blistering diseases. Pemphigus foliaceus (PF) is confined to the skin. Pemphigus vulgaris (PV) and paraneoplastic pemphigus (PNP) can involve the skin and mucosal surfaces of the eyes, mouth, nasopharynx, and oesophagus. Diagnosis is based on clinical findings of epidermal loss, erosions, and superficial blisters of the skin, mucosa, or both. Direct immunofluorescence staining of the skin shows immunoglobulin and C3 deposits on the keratinocyte surface, and histology reveals keratinocyte acantholysis. Treatment aims to reduce or eliminate pathological autoantibodies and consists of long-term immunosuppressive therapy. Outlook for patients with PV and PF is good if the disease is adequately controlled. PNP is associated with malignancy, particularly non-Hodgkin's lymphoma, and mortality can approach 90% due to respiratory failure from bronchiolitis obliterans.
277
Epidemiology of pemphigus
Incidence peak = 50-60 | 1:1
278
Aetiology of pemphigus
Pemphigus vulgaris (PV) and pemphigus foliaceus (PF) are autoimmune, blistering skin diseases that are antibody mediated. Autoantibodies in PV and PF bind exclusively to the desmoglein component of the desmosome. PNP is strongly linked to underlying malignancies. In one third of patients, PNP is a marker for unknown malignancy. The distribution of PNP is bimodal, meaning a disease that occurs in young people, typically associated with non-human herpesvirus 8 Castleman's disease, and in older people with a history of non-Hodgkin's lymphoma, chronic lymphocytic leukaemia, thymoma, and rare sarcomas
279
RFs for pemphigus
STRONG HLA DR4 HLA DQ1 Underlying malignancy (PNP) ``` WEAK ACEi Penicillins Nifedipine Rifampicin ```
280
Sx of pemphigus
``` COMMON Chronic erosive blistering of the skin, mucosa or both Chronic mouth erosions Nikolsky sign Pruritus Conjunctivitis Painful skin Dysphagia ``` PNP: Shortness of breath Painful lips Bloody nose
281
Ix for pemphigus
Skin biopsy + H+E stain Skin biopsy + direct immunofluorescence - staining for IgG, C3, or both in a broad linear band on the surface of epidermal keratinocytes in the suprabasilar region of the epidermis
282
Rx of pemphigus
1. halt progression 2. achieve remission 3. maintain remission ``` Corticosteroids Immunomodulators - MMF, azath, dapsone Rituximab +/- IVIG Plasmapheresis Bone protection due to high dose steroids - calcium carbonate + ergocalciferol + alendronic acid ```
283
Prognosis of pemphigus
The outlook for patients with PV and PF is good if the disease is adequately controlled. There are cases of spontaneous remission, but most patients require some form of long-term immunosuppression. The importance of early and aggressive treatment is supported by the fact that patients with incomplete treatment (i.e., those who have multiple remissions on subtherapeutic therapy) often develop more aggressive disease. In this process, referred to as disease-hardening, more indolent disease becomes resistant to treatment.
284
Complications of pemphigus
``` Volume depletion Infection Pain Malignancy Respiratory failure Conjunctival scarring Steroid osteoporosis Skin dyspigmentation ```
285
A 55-year-old man seeks medical care for superficial erosions and blisters involving the skin of his head, neck, and trunk. Significant involvement of his oral mucosa has made eating difficult. A year ago, he developed tender sores on the buccal mucosa and soft palate of his mouth that were exacerbated by certain foods. His oral disease was initially treated as HSV stomatitis. He was prescribed multiple courses of antibiotics for a presumed superficial skin infection. His condition deteriorated, resulting in the development of eroded areas over his trunk and extremities, with persistent involvement of his scalp and a 10 kg weight loss. The patient notes that the top layer of his skin could be easily removed when firm horizontal pressure was applied. He also reports involvement of his nasal mucosa.
pemphigus Less-common presentations include disease confined exclusively to the scalp, in pemphigus foliaceus (PF), or to the oesophagus or conjunctival mucosa, in pemphigus vulgaris (PV). Vegetative lesions (pemphigus vegetans) are less-common variants of PV in which persistent lesions develop into large fungating nodules. In pemphigus erythematosus (PE), overlap can be seen between PF and lupus. In PE, typical PF lesions occur in sun-exposed areas in patients with underlying lupus. Unusual variants of paraneoplastic pemphigus (PNP) include seronegative lichenoid PNP. Patients develop disease that clinically and pathologically resembles lichen planus as well as PNP, but do not have the classic autoantibodies.
286
A 74-year-old woman with a recent history of non-Hodgkin's lymphoma (NHL) developed severe stomatitis with pronounced involvement of the lips. Her NHL was in remission. She was initially thought to have a variant of erythema multiforme, but an underlying cause such as a drug or upper respiratory infection was not found. The eruption persisted >1 month, which is not typical of erythema multiforme. Over the next few months, she developed superficial skin erosions. Some of the lesions are tender, persistent, target-shaped plaques. She also noted increasing SOB and had recently received antibiotics for a presumed case of pneumonia. Her most significant complaint is intractable stomatitis that was unresponsive to prednisolone, oral antibiotics, and anti-herpetic medications.
pemphigus Less-common presentations include disease confined exclusively to the scalp, in pemphigus foliaceus (PF), or to the oesophagus or conjunctival mucosa, in pemphigus vulgaris (PV). Vegetative lesions (pemphigus vegetans) are less-common variants of PV in which persistent lesions develop into large fungating nodules. In pemphigus erythematosus (PE), overlap can be seen between PF and lupus. In PE, typical PF lesions occur in sun-exposed areas in patients with underlying lupus. Unusual variants of paraneoplastic pemphigus (PNP) include seronegative lichenoid PNP. Patients develop disease that clinically and pathologically resembles lichen planus as well as PNP, but do not have the classic autoantibodies.
287
Define keratocanthoma
Keratoacanthoma is a skin lesion that erupts in sun-damaged skin, rather like a little volcano. It grows for a few months; then it may shrink and resolve by itself. Keratoacanthoma is considered to be a variant of the keratinocyte or non-melanoma skin cancer, squamous cell carcinoma (SCC). As it cannot be clinically reliably distinguished from more severe forms of skin cancer, keratoacanthomas are usually treated surgically. Keratoacanthoma may start at the site of a minor injury to sun-damaged and hair-bearing skin. At first, it may appear as a small pimple or boil and may be squeezed but is found to have a solid core filled with keratin (scale). It then proliferates, and it may be up to 2cm in diameter by the time it is brought to the attention of the doctor.
288
Define Bowens disease
Bowen's disease is essentially equivalent to and used interchangeably with SCC in situ, when not having invaded through the basement membrane.[12] Depending on source, it is classified as precancerous[13] or SCC in situ (technically cancerous but non-invasive).[35][36] In SCC in situ (Bowen's disease), atypical squamous cells proliferate through the whole thickness of the epidermis.[12] The entire tumor is confined to the epidermis and does not invade into the dermis.[12] The cells are often highly atypical under the microscope, and may in fact look more unusual than the cells of some invasive squamous cell carcinomas
289
Define actinic keratosis
Actinic keratosis (AK) lesions are skin-coloured, yellowish, or erythematous, ill-defined, irregularly shaped, small, scaly macules or plaques localised in sun-exposed areas of the body. Typically, they occur in middle-aged or older men with light-coloured skin and a history of chronic sun exposure. They can potentially progress into squamous cell carcinoma (SCC). Chronic keratotic lesions on adult skin that has been chronically exposed to ultraviolet (UV) rays. Has the potential to progress into an invasive squamous cell carcinoma (SCC). Although diagnosed clinically, a biopsy may help to rule out SCC. Treatment consists of destructive methods (e.g., cryotherapy with liquid nitrogen, curettage with or without electrodesiccation, chemical peels, and photodynamic therapy) or topical medication (e.g., topical fluorouracil, imiquimod, diclofenac).
290
Epidemiology of actinic keratosis
The risk increases with advancing age, ranging from 10% prevalence in the third decade of life to >90% in people older than 80 years. Prevalence in males is higher than in females in the US (26.5% in males versus 10.2% in females), UK (15% in males versus 6% in females), and Australia (55% in males versus 37% in females). People with light-coloured skin are 6 times more likely than people with darker skin types to develop AKs.
291
Aetiology of actinic keratosis
Chronic exposure to UV rays, mostly UVB (290 to 320 nanometres) is a key aetiological factor resulting in damage to keratinocyte DNA and has been implicated in skin carcinogenesis. Other factors include chronic exposure to UVA, X-rays, radioisotopes, arsenic, and human papillomavirus (particularly in immunocompromised and organ-transplanted patients, and in patients with epidermodysplasia verruciformis).
292
RFs for actinic keratosis
``` STRONG Chronic UVB Fitzpatrick 1/2 Age >40 Male sex Immunocompromise Xeroderma pigmentosum ```
293
Sx of actinic keratosis
COMMON Single or multiple scaly macule or papules Scaly lesions with hyperkeratotic surface Well-defined, scaly, brown lesions Lesions resembling seborrheic keratosis / melanoma Hypertrophic conical shaped protuberances growing from the surface of the skin Scaly red roughened with induration, fissuring and ulceration of the lower lip to the commissures Lesion on sun-exposed are of the body UNCOMMON Skin-coloured papillomatous wart like papules Plaques with very mild scale Pruritus or bleeding
294
Ix for actinic keratosis
Dermoscopy - strawberry pattern (erythematous background, pink-to-red pseudo-network, thin undulated vessels surrounding hair follicles, white-to-yellow scale, hair follicles filled with yellowish keratotic plugs, surrounded by a white halo). Possible early signs of progression of facial AKs to squamous cell carcinoma (SCC) include dotted vessels surrounding hair follicles, starburst pattern, and coalescence of keratotic follicles Skin biopsy - histological changes of intra-epidermal keratinocytic dysplasia
295
Rx of actinic keratosis
1. Cryosurgery 2. Imiquimod topical or diclofenac 3. Chemical peels 4. Photodynamic therapy 5. Curettage + electrodesiccation THICK LESIONS 1. Cryosurgery 2. Dermabrasion
296
Prognosis of actinic keratosis
The calculated lifetime risk of malignant transformation for a patient with AKs followed up for 10 years is between 6.1% and 10.2%. In addition, one study showed that 40% of all SCCs developed from clinically normal skin in the prior year, and 60% of SCC arose from previous AKs.
297
Complications of actinic keratosis
``` Bleeding Secondary infection Yes related to Rx Progression to bowen's Progression to invasive SCC ```
298
A 65-year-old man with light-coloured skin presents with asymptomatic pink and yellowish scaly, irregularly shaped, rough papules and macules. The lesions are localised on the scalp, forehead, ears, dorsal forearms, and dorsal aspect of hands, over chronically sun damaged-looking skin. He has been working outdoors for many years without wearing any sunscreen or protective clothing. He has had previous treatment with curettage, cryotherapy, and topical fluorouracil. He is presenting with new lesions, and is concerned about developing skin cancer.
actinic keratosis Other clinical presentations include: scaly lesions with a hyperkeratotic surface (hyperkeratotic AKs); well-defined, scaly, brown lesions resembling solar lentigo (pigmented AKs);[1] lesions resembling seborrhoeic keratosis, melanocytic naevus, and early malignant melanoma (spreading pigmented AKs);[5] skin-coloured, papillomatous, elevated wart-like papules (verrucous AKs); plaques with very mild scale over very thin shiny skin (atrophic AKs); violaceous well-defined papules with fine white lines on the surface (lichen planus-like or lichenoid AKs); hypertrophic conical-shaped protuberances growing from the surface of the skin (cutaneous horn); and scaly red roughness with induration, fissuring, and ulceration of the lower lip to the commissures (actinic cheilitis).[1][4]
299
Rx of pressure ulcer
Treatment for all patients should include pressure relief, good hygiene practice, and skin care, particularly in the sacral region. Appropriate pressure-reducing aids, including mattresses and wheelchair or seat cushions, should be immediately provided. Little evidence supports the use of a specific support surface over other alternatives. Analgesia Dietary optimisation - additional protein or amino acid supplementation can enhance healing Systemic antibiotic therapy is required for patients with bacteraemia, sepsis, advancing cellulitis, or osteomyelitis. Systemic antibiotics are not required for pressure ulcers that exhibit only signs of local infection. Debridement of necrotic tissue Maggots, water jet, enzymatic agents DEEP ULCERS GRADE 3 + 4 Surgical debridement + reconstruction with flap
300
Prognosis of pressure ulcer
Spinal injury patients: assuming a patient is otherwise in good health, prognosis is good, as long as appropriate treatment is provided promptly; however, these patients are at high risk of recurrence. Seriously ill patients with extensive ulcers: for patients who have other serious medical problems, the prognosis is poor, as they may be unable to tolerate or accept the intensive treatment required to close their wound surgically. Infirm patients with superficial damage: prognosis is reasonable as long as appropriate local wound care is provided.
301
Complications of pressure ulcer
Sepsis Cellulitis Osteomyelitis Mortality
302
A 65-year-old man presents with fever and respiratory distress. He rapidly develops progressive hypoxia and hypotension requiring ventilator support and pressors. Efforts to reposition the patient result in marked desaturations. One week later, following stabilisation of his medical condition, he is noted to have an extensive area of tissue damage over his sacral region. The damage around the margin of the wound appears relatively superficial, with some signs of partial skin loss. Towards the centre, directly over the sacrum, is an area of necrotic tissue, which is starting to separate spontaneously, exposing a cavity containing viscous yellow slough. A purulent discharge is draining from this cavity.
pressure ulcer Pressure ulcers, while typically thought of as a condition of frail elderly patients, may occur among patients of any age and in any setting. When not located over a bony prominence, pressure ulcers are usually the result of an external device. Examples include oxygen tubing causing a pressure ulcer on the ear or a lower leg cast causing an ulcer on the leg. External devices may also cause mucosal pressure injury.
303
An 80-year-old woman in a residential care facility, who has recently suffered a stroke with a resulting right hemiplegia, presents with a painful area on right heel. On examination the heel is found to be covered with a layer of hard, black, necrotic tissue with a leathery appearance.
pressure ulcer Pressure ulcers, while typically thought of as a condition of frail elderly patients, may occur among patients of any age and in any setting. When not located over a bony prominence, pressure ulcers are usually the result of an external device. Examples include oxygen tubing causing a pressure ulcer on the ear or a lower leg cast causing an ulcer on the leg. External devices may also cause mucosal pressure injury.
304
Define psoriasis
Psoriasis is a chronic inflammatory skin disease characterised by erythematous, circumscribed scaly papules, and plaques. It can cause itching, irritation, burning, and stinging. Although the classification typically includes psoriatic arthritis, skin manifestations only are discussed in this topic. Lesions are red, inflamed, silvery-white scaly, and circumscribed papules and plaques; often affecting elbows, knees, extensor limbs, and scalp, and, less commonly, nails, ear, and umbilical region. Typically lifelong, with a fluctuating course of exacerbations and remission of lesions, which may be aggravated by genetic, infectious, emotional, and environmental factors. Diagnosis is usually clinical. Mild or limited psoriasis is treated with topical corticosteroids and/or vitamin D analogues. Moderate to severe and/or extensive psoriasis may require phototherapy, and systemic agents such as oral retinoids, methotrexate, biological agents, ciclosporin or apremilast.
305
Epidemiology of psoriasis
Mean age of onset is 28 years Prevalence is between 1.5% and 5.0%
306
Aetiology of psoriasis
Genetics Immunology Infections
307
RFs for psoriasis
STRONG Genetic Infection Local trauma ``` WEAK Stress Smoking Light skin Alcohol intake ```
308
Sx of psoriasis
Typically erythematous, circumscribed scaly papules and plaques on elbows, knees, extensor surfaces of limbs, scalp, and, less commonly, nails, ears, and umbilical region. In plaque psoriasis, there are raised inflamed plaque lesions with a superficial silvery-white scaly eruption. The scale may be scraped away to reveal inflamed and sometimes friable skin beneath. Pinpoint bleeding points are known as Auspitz's sign. In guttate psoriasis, there are widespread, erythematous, fine, scaly papules (water drop appearance) on trunk, arms, and legs. The lesions often erupt after an upper respiratory infection. In pustular psoriasis, acute generalised pustular psoriasis (von Zumbusch) is rare, severe, and urgent; palmoplantar pustulosis affects palms and soles and is chronic. In erythroderma (erythrodermic psoriasis), there is generalised erythema with fine scaling. It is often associated with pain, irritation, and sometimes severe itching. Itching and pain from the psoriatic lesions can occur, especially in those with moderate to severe psoriasis. Bleeding may occur if the lesions are scratched.
309
Ix for psoriasis
Clinical Possible to do skin biopsy
310
Rx of psoriasis
1. Topical corticosteroid or topical vitamin D analogue Mod-Severe 1. Phototherapy 1. Methrotrexate 1. Apremilast 1. Biological - infliximab, ustekinumab, adalimumab, etanercept, secukinumab 1. Oral retinoid 2. Ciclosporin Guttate = same Rx but possible to use retinoids Pustular = oral retinoid, re-puva
311
Prognosis of psoriasis
The exact natural history of psoriasis is poorly understood. However, psoriasis is generally considered a chronic disease with a fluctuating course. Long-term control with topical and/or systemic medications is necessary for many patients.
312
Complications of psoriasis
``` CV complications Psoriatic arthritis Depression Lymphoma Secondary infection ``` Patients with psoriasis or psoriatic arthritis have an increased incidence of cardiovascular disease (e.g., myocardial infarction and stroke)
313
A middle-aged man with a known history of psoriasis presents with white scaly papules and plaques on his elbows, extensor arms, knees, and shins. In the past 6 months, these lesions have become much worse and have started to appear on his waist and hip. Scaly and flaky eruptions are also present on his scalp, ears, and eyebrows. He describes the lesions as being itchy and irritating. He is a heavy smoker and has been unsuccessful in a previous attempt at smoking cessation.
psoriasis Psoriasis can present in atypical areas, such as the scalp, palms and soles, digits, nail, penis, nappy area, gluteal cleft, skin folds, or oral mucosa. Pustular psoriasis presents as sudden-onset disease with generalised pustulosis. However, the pustules carry no bacteria and patients are not febrile. Psoriatic arthritis is often insidious, with stiffness and inflammation around finger and toe joints only. The associated cutaneous lesions may be very minor, which makes it difficult to establish the diagnosis of psoriasis.
314
A young man without a known history of psoriasis or skin disorder had a sudden onset of wide-spreading, white-scaly, oval- to round-shaped erythematous papules, which have been present for 2 weeks. Lesions are primarily on his trunk but also appear scattered on his arms and legs. He recalls a recent episode of sore throat and upper respiratory tract infection. A short course of antibiotics seemed to help, but did not clear the lesions.
psoriasis Psoriasis can present in atypical areas, such as the scalp, palms and soles, digits, nail, penis, nappy area, gluteal cleft, skin folds, or oral mucosa. Pustular psoriasis presents as sudden-onset disease with generalised pustulosis. However, the pustules carry no bacteria and patients are not febrile. Psoriatic arthritis is often insidious, with stiffness and inflammation around finger and toe joints only. The associated cutaneous lesions may be very minor, which makes it difficult to establish the diagnosis of psoriasis.
315
Define SCC
Cutaneous squamous cell carcinoma (SCC) is the proliferation of atypical, transformed keratinocytes in the skin with malignant behaviour. It ranges from in situ tumours (also known as Bowen's disease) to invasive tumours and metastatic disease. Malignant tumour of keratinocytes arising in the epidermis of the skin. Second most common non-melanoma skin cancer worldwide, secondary to basal cell carcinoma. Cumulative ultraviolet exposure, most commonly from the sun, and immunosuppression are major risk factors. Treatments include non-surgical destruction (e.g., using cryotherapy), topical chemotherapy, traditional surgical excision, and Mohs micrographic surgery.
316
Epidemiology of SCC
SCCs are most frequently observed in photoexposed skin, often in those >40 years of age Men have a higher incidence than women, and those with lighter skin types are at increased risk
317
Aetiology of SCC
Similar to pre-malignant skin tumours such as actinic keratoses, the incidence of non-melanoma skin tumours varies dramatically depending on skin phototype or constitutive pigmentation, cumulative sun exposure, and geographic latitude. In addition to solar UV exposure, other factors are known to increase the risk of SCC, such as ionising radiation, burns, previous psoralen and UV-A light therapy, hereditary skin conditions, environmental toxins such as arsenic and tar, human papillomavirus, and some immunocompromised states. SCC arising only in the red inked areas within a multi-coloured tattoo has been described, suggesting that red tattoo ink in the skin may be an infrequent risk factor for developing SCC
318
RFs for SCC
``` STRONG UV radiation exposure Immunosuppression Fair skin Xeroderma Older age Male sex Ionising radiation Carcinogens Actinic keratosis Previous SCC ``` ``` WEAK HPV Tobacco smoking Thiazide diuretics Tattoos ```
319
Sx of SCC
COMMON ``` GROWTH BLEEDING SUN EXPOSED Tender or itchy [marjolin ulcer] Erythematous Thin, flesh coloured plaques (Bowens) Dome shaped nodule with rapid growth (dome-shaped nodule) Exophytic, fungating, verrucous nodules ``` UNCOMMON ``` Ulcerated tumours Lymphadenopathy Bone pain Hepatomegaly Neurological signs ```
320
Ix for SCC
Biopsy - full thickness keratinocyte atypia CT/MRI/PET - lymphadenopathy and/or visceral nodules suggestive of metastases CXR + LFTs - mets
321
Rx of SCC
``` SCC in situ 1. Destructive therapies: cryotherapy OR electrodessication/curettage OR photodynamic therapy 1. fluorouracil topical: (5%) apply to the affected area(s) twice daily for 3-6 weeks OR imiquimod topical: (5%) apply to the affected area(s) once daily, 2-3 times weekly for 3-6 weeks 2. Mohs Surgery (microscopically controlled surgery with frozen section histology) or conventional surgical excision +/- radiotherapy ``` INVASIVE MOHS or conventional surgical excision ``` METASTATIC Same With radiotherapy OR chemo: capecitabine and interferon alfa 2b ```
322
Prognosis of SCC
Prognosis depends on the depth of tumour invasion, histological pattern, and immunological status of the patient. Generally the metastatic potential of SCC is low, at 3% Metastases are more common in tumours of the ear (10%) or lip (30%), scalp, and extremities, and especially in patients who have received immunosuppressive therapy. For those with metastatic disease, the long-term prognosis is extremely poor. Ten-year survival rates are <20% for patients with regional lymph node involvement and <10% for patients with distant metastases.
323
Complications of SCC
Scarring | Deformities